You are on page 1of 122

Objetivo General.

1. Compilar la teora bsica en un nico documento destinado al estudiante de


fsica, matemtica o ciencias en general; como apoyo bibliogrco para
cursos sobre teora de nmeros.

Objetivos Especcos.

1. Reunir en un nico tratado los tpicos fundamentales y recurrentes en los


programas de teora de nmeros, con la nalidad de dirigirlo a estudiantes
como material bibliogrco, para apoyo de la asignatura y consulta.
2. Mostrar en un lenguaje sencillo los principales resultados de la teora de
nmeros, acompaado de buena ejercitacin, adems de algunas aplica-
ciones y actualizaciones en este campo.

1
Introduccin

La teora de nmeros o aritmtica como tambin es llamada es quizs junto


con la geometra la rama de la matemtica ms antigua, pero a diferencia de esta
en la cual pueden recrearse formas y cuerpos para ser estudiados, la aritmtica
suele ser rida, abstracta y desprovista de atractivo para casi cualquier lector,
muchos matemticos han llamado a la teora de nmeros como la rama ms
difcil de esta ciencia, otros le han dado el ttulo de Reina de las matemticas,
sea cual sea el calicativo la aritmtica siempre ha estado rodeada de un aura de
misticismo y escepticismo para el lector. A pesar de que esta rama cuenta con
un gran campo de aplicacin en disciplinas como la computacin, criptografa,
nanzas, biologa, fsica y las matemticas mismas. Sin mencionar que ha en-
gendrado los problemas ms famosos y difciles de las matemticas, algunos aun
sin solucin en la actualidad; as tambin ha dado lugar a la creacin de nuevas y
modernas ramas de las matemticas tales como: la teora analtica de nmeros,
la teora algebraica de nmeros, teora de curvas elpticas, entre otras.

La teora de nmeros no es propia de un nivel particular de educacin,


podemos encontrar tpicos de sta desde la escuela primaria hasta la univer-
sidad. Propiamente dicho, en nuestro pas se forman profesionales en educacin
con mencin en matemticas donde deben estudiar teora de nmeros. Es pre-
cisamente por estos ltimos que escribimos este trabajo.

Por las caractersticas socioculturales y econmicas de nuestro pas, es difcil


acceder a bibliografa actualizada y adecuada para ciertos niveles educativos y
para determinados nes acadmicos, motivados por esta causa hemos decidido
escribir este trabajo compilatorio en su gran medida, pero con las particulari-
dades de: mostrar la teora expuesta con una claridad de lenguaje y explicacin
paso a paso, presentar una variedad de ejercicios resueltos y otra gama de ejer-
cicios propuestos con su respuesta o sugerencias para su solucin, un material
autosuciente en el sentido que cada captulo dota de lo necesario para el sigu-
iente sin la necesidad de recurrir a otros medios y nalmente las aplicaciones a
otras ciencias o dentro de las matemticas mismas y los resultados ms recientes
en esta rama.

La intencin de este documento es crear, no un recetario sino, un medio


didctico-tcnico, dirigido a estudiantes de nivel universitario que tengan que
enfrentarse a un curso de teora de nmeros, pues aqu podrn adquirir una
formacin terica-prctica en cuanto a conocimiento terico y estrategia para
la solucin de problemas.

2
Resea Histrica De La Teora De Nmeros.

Construyendo el nmero

Hoy en da encontramos nmeros en casi cualquier disciplina cientca, tec-


nologa o incluso en el ambiente ordinario, pero este conjunto de grafos son el
resultado de mucho tiempo de evolucin y complejas relaciones culturales que
datan desde tiempos del origen de la humanidad misma.

El concepto de nmero surgi como consecuencia de la necesidad prctica


de contar los objetos. Inicialmente se contaba con ayuda de los medios que
se dispona: dedos, piedras, conos de abetos, etc. Huellas de estos se han
conservado en las denominaciones de los clculos matemticos: Por ejemplo,
calculus en su traduccin del latn signica cuenta con piedras. Debido a los
medios utilizados para contar, la serie natural se conceba nita y se contaba de
5 en 5 (para el caso de los dedos de las manos) y luego se iniciaba nuevamente
la cuenta formando paquetes de 5. Presumiblemente esta sea la razn de tener
un sistema decimal por poseer 10 dedos en las manos utilizados como medios
de clculo.

Junto a la utilizacin de ms y ms nmeros surgieron y se desarrollaron sus


smbolos, y los propios nmeros formaron sistemas. Para los primeros perodos
de la historia de la humanidad es caracterstico encontrarse con una diversidad
de sistemas numricos. Que paulatinamente se perfeccionaron y unicaron como
consecuencia de las interacciones culturales entre las distintas razas.

Algunos ejemplos se muestran en las guras siguientes:

Numeracin Egipcia

3
Numeracin Eslava

Nuestros numerales (0, 1, 2, 3, 4, 5, 6, 7 ,8 ,9) se suelen llamar rabes o


arbigos a pesar que se parecen muy poco a los utilizados en los pases de la
regin rabe como Egipto, Irak, Siria, Arabia, etc. Pero esta denominacin
de nmeros arbigos se debe a que los principios en los que se basan los dos
sistemas es el mismo y a que los signos usados pueden haberse derivado de los
rabes.

Con esta universalizacin del sistema numrico, la conciencia del nmero se


volvi lo sucientemente extendida y clara como para llegar al punto de sentir
la necesidad de expresar esta propiedad de alguna manera, al inicio presumible-
mente solo en un lenguaje simblico.

4
Nacimiento De La Aritmtica

La aritmtica de la traduccin griega arihtmos o nmero tiene su origen en el


misticismo numerolgico de los griegos o ms especcamente de la escuela de los
pitagricos. Los pitagricos fueron con mucha seguridad los primeros en estudiar
algunas de las propiedades de los nmeros y a dar las primeras clasicaciones
como pares, impares, primos, compuestos, nmeros gurados, entre otros.

En la poca moderna la Aritmtica o Teora de nmeros como tambin es


llamada se concibe como la rama de las matemticas encargada de estudiar las
propiedades de los nmeros naturales (0, 1, 2,. . . ) o enteros (. . . ,-2, -1, 0, 1,
2,. . . ). Entre las propiedades de mayor inters se cuentan: la divisibilidad y
cuando un nmero es primo o compuesto.

La poca de Euclides

Cercano al 300 a .c ocurri uno de los principales sucesos para la historia de


las matemticas, la aparicin de los Elementos de Euclides obra monumental de
13 libros que recoge el conocimiento matemtico alcanzado hasta la poca, que
sacara a la aritmtica de la numerologa y del misticismo, para convertirla en
una ciencia estricta y deductiva. Existe la concepcin errnea que los elementos
es una obra enteramente dedicada a la geometra, los libros VII, VIII y IX estn
dedicados enteramente a la teora de nmeros.

El libro VII comienza con dos proposiciones que constituyen lo que hoy
conocemos como el algoritmo de Euclides para obtener el mximo comn divisor
de dos nmeros dados. El libro VIII trata las propiedades de los cuadrados y los
cubos. Y por ltimo y el que posee un inters especial el libro IX, que contiene el
teorema y la demostracin del mismo sobre la innitud de los primos. Tambin
atac el problema de los nmeros perfectos, dando la demostracin de que todo
nmero perfecto es de la forma 2p 1 (2p 1),en donde p y 2p 1 son primos. Dos
mil aos ms tarde Euler demostr el recproco del teorema diciendo que todo
nmero par perfecto debe ser del tipo descrito por Euclides.

Por ejemplo. Para 6 tenemos:

6 = 22 1
22 1 =2 3

Todo nmero de la forma 2p 1 , en donde p es primo se conocen como nmeros


de Mersenne, que los estudi en 1644. An hoy no se conoce si existen nmeros
perfectos impares.

La Aritmtica De Diofanto

Los 250 aos que siguieron a la desaparicin de Euclides la teora de nmeros


entr en un periodo de oscuridad hasta la llegada de Diofanto de Alejandra,
quin public 13 libros, de los cuales se han conservado 6.

5
La aritmtica de Diofanto en lo que ha llegado hasta nosotros, est dedicada
casi completamente a la resolucin exacta de ecuaciones determinadas e inde-
terminadas. Una de los principales aportes de Diofanto a las matemticas fue
haber introducido la notacin algebraica que precedi a lo que hoy utilizamos.
La arithmetica no consiste en exposicin sistemtica sobre las operaciones o
funciones algebraicas, sino en una coleccin de 150 problemas, resueltos todos
en trminos de ejemplos numricos completos y especcos.

Diofanto fue hbil para resolver ecuaciones algebraicas con dos o tres incg-
nitas. Muchos de estos problemas se originaron en la teora de nmeros y a l le
pareci natural encontrar soluciones enteras. Las ecuaciones que deben ser re-
sueltas por medio de valores enteros reciben el nombre de ecuaciones difnticas
y el estudio de tales ecuaciones anlisis diofntico.

Pierre de Fermat y La Teora De Nmeros

El trabajo de Diofanto encuentra en el siglo diecisiete a su mejor intrprete,


Pierre de Fermat quien se convertira en el creador de la moderna teora de
nmeros.

Dentro de los resultados obtenidos por Fermat tenemos que armaba que
n
los nmeros de la forma fn = 22 + 1 es un nmero primo 8n 2 N, pero esta
conjetura resulto errnea, Euler demostrara que para n = 5 es compuesto.

El conocido teorema de Pitgoras, inspir el resultado ms famoso y quizs


el ms difcil planteado por Fermat. El llamado ltimo teorema de Fermat, la
ecuacin diofntica x2 + y 2 = z 2 ; 8n > 2 , no tiene solucin en los nmeros
enteros. Este fue un problema que se resisti 350 aos a las mejores mentes
matemticas hasta que un joven matemtico ingls, Andrew Wiles, en mayo
de 1995 publicar un artculo de 130 pginas en Annals of Mathematics con la
solucin al problema.

Aunque tal vez menos impresionante, pero otra gran paso en la teora de
nmeros debido a Fermat es el pequeo teorema que lleva su nombre: si p es
primo positivo y a es un entero coprimo de p, entonces p divide a, ap 1 1
ap 1 1 mod p , cuya demostracin se la debemos a Euler.

Fermat logr verdaderos y muy profundos avances en esta rama de las


matemticas, a pesar que su forma de razonar era muy intuitiva y casi nunca
daba una demostracin general, sin embargo dio algunas muy interesantes para
los siguientes teoremas:

Todo nmero entero o es un nmero triangular o una suma de 2 3 nmeros


triangulares; todo entero o es cuadrtico o suma de 2, 3, 4 cuadrticos; todo
nmero o es pentagonal o es suma de 2, 3, 4 5 pentagonales, y as sucesiva-
mente.

6
Tambin se le debe el resultado de que todo nmero primo de la forma 4n+1,
es suma de dos cuadrados.

Por ejemplo.

5 = 12 + 22 ; 13 = 22 + 32 ; 17 = 12 + 42

Un nuevo rumbo para La Aritmtica

Posterior al trabajo de Fermat, resuenan los nombres de las que quizs sean
las guras ms representativas de la moderna teora de nmeros: Leonard Euler
(1707-1783), Lagrange (1763-1813), K.F Gauss ( 1777-1855) y Dirichlet (1805-
1859).

Gracias a Euler se vericaron o refutaron algunos de los ya mencionados


problemas planteados por Fermat. Dictada por Euler, ya ciego, alrededor del
ao 1767, la Aritmtica Universal apareci, obra monogrca que consta de dos
partes; en los tres pargrafos de la primera parte dirigi una atencin especial
a las reglas de resolucin de problemas aritmticos y al desarrollo del aparato
simblico-lingstico del lgebra. El ltimo pargrafo incluye preferentemente
los mtodos para buscar soluciones enteras de las ecuaciones indeterminadas
de primer grado y grados superiores. Aqu se le agrega la resolucin del gran
teorema de Fermat para n = 3 y n = 4:

Todo hace indicar que la motivacin especial por la teora de nmeros,


provino de la correspondencia mantenida entre Euler y Goldbach otro brillante
matemtico. En una de estas cartas, fechada de 1 de diciembre de 1729, Gold-
bach pregunta a Euler si conoce el resultado donde Fermat arma que todo
n
nmero de la forma fn = 22 + 1 es primo, Euler contesta con un contra ejem-
25
plo, f5 = 2 + 1 = 4294967297; el cual es divisible por 641 refutando as el
resultado de Fermat.

Adems de este hecho demostr el pequeo teorema de Fermat, introdujo


la funcin '(m); cuyos valores son iguales a la cantidad de nmeros menores
que m y que son coprimos con l, en 1722 cre la ley de reciprocidad cuadrtica
y todo lo concerniente al problema de la representacin de nmeros en formas
cuadrticas.

Los trabajos de Euler determinaron la problemtica, la estructura, y los


mtodos de la teora algebraica de los nmeros.

7
Despus de Euler, la ley de reciprocidad cuadrtica la demostr Legendre
(dio una demostracin incompleta). Gauss, hasta el ao 1801 dio ocho demostra-
ciones de esta ley. No obstante Legendre en 1880 encontr la forma ms cmoda
de escribir esta ley:
p q p 1 q 1
;
= ( 1) 2 2
q p

Entre los problemas aditivos de la teora de nmeros, propuestos en el siglo


XVIII, se encuentra tambin el problema de Waring (1770), cualquier nmero
natural n 2 es representable como la suma de n esimas potencias de nmeros
naturales, adems el nmero r de trminos de la suma depende solo de n. Waring
no dio su demostracin. Como en la mayora de los problemas de la teora de
nmeros, el xito se logr con mucha dicultad. As, Lagrange demostr que
si n = 2, entonces r = 4: Posterior a esto se encontraron otros resultados
particulares hasta que en 1909, Hilbert dio la primera demostracin general.

En los trabajos de Euler, Lagrange, Legendre, Lambert y otros matemticos,


fueron elaborados y renados numerosos e ingeniosos mtodos de la teora de
nmeros, tanto algebraicos como analticos. Todas estas investigaciones, natu-
ralmente, necesitaban sistematizacin, reduccin a una estructura lgica y bien
estructurada de manera original. Esta dura tarea fue iniciada por Legendre en
los aos de 1797-1798 que titul Experiencia de la teora de nmeros, teniendo
como objetivo construir un sistema de resultados sobre las propiedades de los
nmeros enteros. En posteriores ediciones fue completado con el trabajo de
Gauss, Abel y otros brillantes matemticos del siglo XIX. En esta presentacin
est contenida el enorme cmulo de conocimiento sobre teora de nmeros, lo
que le da un signicado histrico y un signicado intelectual invaluable como
gua para iniciar el camino de los nmeros.

8
1 Nmeros Enteros Naturales

"Dios hizo los nmeros naturales; el resto es obra del hombre"


Leopold Kronecker.

En este captulo abordaremos los principios fundamentales que rigen a los


nmeros naturales, El principio del buen orden y el Principio de Induccin
matemtica. Adems de estudiar el sistema de los naturales desde la ptica
axiomtica de Peano.

1.1 Principio del Buen Orden (PBO)

1.1.1 Denicin. Decimos que a es mayor que b (simbolizado a > b), si la


ecuacin b + x = a es soluble para algn nmero natural x.(para el caso
x = 0, se obtiene la igualdad b = a)
La relacin a es mayor que b, puede expresarse equivalentemente as: b < a

La relacin "mayor que" antes denida tiene las siguientes propiedades:

1. Tricotoma. Una y solamente una de las relaciones siguientes debe cumplirse:


a > b; a = b; a < b

Demostracin.
Supongamos que se cumplen a > b y a = b: De la denicin de mayor,
tenemos la existencia de algn x natural, tal que a = b + x, por hiptesis
y transitividad se sigue que b = b + x, lo cual es absurdo. Ahora supongamos
como cierto que a < b y a = b; anlogamente, se tiene un x tal que b = a+x, por
hiptesis y transitividad de la igualdad pasa que a = a + x; lo que nuevamente
es absurdo. As, puede concluirse que solo una de las relaciones puede cumplirse

2. Propiedad transitiva.
Si a > b y b > c; entonces a > c

Demostracin. Si a > b y b > c, esto implica la existencia de naturales


x y y tales que a = b + x y b = c + y: Podemos entonces reemplazar b por
c + y en la primera ecuacin as: a = (c + y) + x = c + (y + x) ; por propiedad
asociativa. Sin embargo (y + x) = z es un nmero natural por cerradura de la
adicin. Esto prueba que a = c + z y por tanto a > c

9
3. Si a > b; entonces a + c > b + c

Demostracin.
Para realizar esta demostracin empezaremos por escribirla en la forma sim-
blica:
Si a > b =) a + c > b + c
Tomando su contrarrecproco

(a + c > b + c ) =) (a > b)

que equivalentemente es

a+c b + c =) a b

Aqu el smbolo lo utilizamos a n de simplicar la escritura de " < = "

a+c<b+c =) a+c+x=b+c def. mayor que


i. =) a+x=b prop. cancelativa
=) a<b def. mayor que
ii. Si a + c = b + c; entonces a = b por la propiedad cancelativa.

La demostracin de la propiedad nmero 3 se realiz en la base de la ley


cancelativa, de la cual se dar una demostracin ms adelante.

Principio de Buen Orden (PBO): Cualquier conjunto de nmeros nat-


urales que contenga al menos un elemento, contiene un elemento mnimo.

1.1.1 Teorema. No hay ningn natural entre 0 y 1.

Demostracin.
Supongamos que existe un nmero a; con la siguiente propiedad, 0 < a < 1.
Entonces, existe un conjunto A no vaco de elementos menores que 1: Luego,
por PBO, A tiene elemento mnimo, llammosle m; ser tal que 0 < m < 1:
Mutiplicando toda la desigualdad por m, 0 < m2 < m: Entonces m2 es otro
natural del conjunto A, menor que el supuesto elemento mnimo de A. Esta
contradiccin demuestra el teorema.

1.1.2 Teorema : Un conjunto H de enteros naturales que incluya al 1 y que


incluya al n + 1 siempre que incluya al n; incluye tambin a cualquier
entero natural.

10
Demostracin.
La prueba consiste en demostrar que el conjunto H 0 de los naturales que
no estn en H es vaco, esto es H 0 = fx 2 N : x 2 = Hg = : Supongamos lo
contrario, esto es H 0 contiene al menos un elemento; luego por PBO, H 0 tiene
elemento mnimo m: Pero m 6= 1; por hiptesis 1 2 H; luego por el teorema
anterior m > 1; y m 1 > 1: Adems 1 < m 1 < m; resulta que, como m
es elemento mnimo de H 0 ; m 1 debe estar en H. Segn la hiptesis puede
deducirse que (m 1) + 1 = m 2 H. Lo que contradice nuestro supuesto.

1.2 Principio de Induccin Matemtica (PIM)

1.2.1 Teorema. Principio de induccin Completa. Asociemos a cada


nmero natural n una proposicin P (n), la cual puede ser verdadera o
falsa. Si, primero, P (1) es verdadera y, segundo, para cualquier k la
verdad de P (k) implica la de P (k + 1) ; entonces P (n) es verdadera para
todo natural n:

Demostracin.
Una proposicin P (n) ; es vlida para todo nmero natural n si:

1. Es vlida para n = 1
2. De su validez para un nmero natural cualquiera n = k se deduce su
validez para n = k + 1

Supongamos lo contrario, es decir, que la proposicin no es vlida para


cualquier nmero natural n: Entonces, existe un nmero natural n0 tal que,
la proposicin es falsa; por la condicin 1. n0 6= 1: Luego, existe un nmero
natural n1 tal que n0 = n1 + 1: Armamos que

n0 > n1 y Pn1 es falsa:


Porque n1 < n1 + 1 = n0 y si Pn1 es verdadera, entonces por la condicin 2
la proposicin Pn1 +1 = Pn0 sera verdadera, lo cual contradice nuestro supuesto.
(existe un nmero natural n0 tal que, la proposicin es falsa). Aplicando a n1 el
mismo razonamiento que a n0 ; encontramos que existe un nmero natural n2 ;
tal que

n0 > n1 > n2 y Pn2 es falsa


continuando de esta manera obtenemos una sucesin innita decreciente de
nmeros naturales

n0 > n1 > n2 > n3 > > ni >

11
Lo cual es imposible puesto que cualquier subconjunto de nmeros naturales
tiene elemento mnimo (PBO).

La demostracin se ha podido completar en la base del PBO, as mismo,


tomando como fundamento el PIM puede probarse el PBO.

Ejemplos de algunas demostraciones usando PIM

1. Demustrese que para todo natural n, la suma de los primeros n trminos es

n (n + 1)
2
Demostracin.

n (n + 1)
1+2+3+ +n=
2
Primeramente vericamos el caso particular n = 1, esto es

1 (1 + 1) 2
1= = =1
2 2
Por tanto, P1 es vlida.

Ahora, supongamos que Pk es vlida. Entonces la hiptesis de induccin es:

k (k + 1)
1+2+3+ +k =
2
Nuestro objetivo es demostrar la validez de Pk+1 ; esto es

(k + 1) [(k + 1) + 1]
1+2+3+ + (k + 1) =
2
Reescribiendo el primer trmino de la igualdad y aplicando la hiptesis induc-
tiva; como sigue:

(1 + 2 + 3 + + k) + k + 1 Agrupamos los primeros k trminos


k(k+1)
(1 + 2 + 3 + + k) + (k + 1) = 2 + (k + 1) hiptesis de induccin
k(k+1)+2(k+1)
(1 + 2 + 3 + + k) + (k + 1) = 2 Suma de naturales
(K+1)(k+2)
(1 + +3 + + k) + (k + 1) = 2 P. Distributiva
(k+1)[(k+1)+1]
(1 + 2 + 3 + + k) + (k + 1) = 2 k + 2 = (k + 1) + 1

Con lo que se demuestra que Pk+1 es cierta.

12
2. Demustre la validez de la siguiente armacin:

2+4+6+ + 2n = n(n + 1)

Demostracin.
Para el caso particular n = 1

2 = 1(1 + 1) = 2
por tanto la proposicin es verdadera para n = 1

Suponiendo la validez de la expresin para algn k 2 N. La hiptesis resulta

2+4+6+ + 2k = k(k + 1)
As, la tesis a probar es

2+4+6+ + 2 (k + 1) = (k + 1) [(k + 1) + 1]
Luego

(2 + 4 + 6 + + 2k) + 2 (k + 1) Agrupamos los primeros k trminos


(2 + 4 + 6 + + 2k) + 2 (k + 1) = k(k + 1) + 2 (k + 1) hiptesis de induccin
(2 + 4 + 6 + + 2k) + 2 (k + 1) = (k + 1) (k + 2) P. Distributiva
(2 + 4 + 6 + + 2k) + 2 (k + 1) = (k + 1) [(k + 1) + 1] k + 2 = (k + 1) + 1

Con lo que se demuestra que Pk+1 es cierta.

1.3 Nmeros Naturales y Axiomas de Peano

Se dene el conjunto N de los nmeros naturales como un conjunto que verica


los cinco axiomas siguientes:

1. Existe un elemento de N al que llamaremos cero (0), esto es,

02N

2. Existe la aplicacin siguiente, sig : N ! N :

sig : N ! N; 8n 2 N; sig (n) 2 N

3. El cero no es imagen por la aplicacin siguiente:

8n 2 N; sig (n) 6= 0

13
4. La aplicacin siguiente es inyectiva:

8n; m 2 N; si sig (n) = sig (m) =) n = m

5. Se verica la induccin completa: Si S N y satisface

02S
=) S = N
8n 2 S =) sig (n) 2 S

A partir de estos cinco axiomas, y usando sistemticamente la induccin


completa, podemos probar todas las propiedades del conjunto N.

1.3.1 Denicin. Denimos la suma de nmeros naturales como una apli-


cacin + : N N ! N; de forma tal que (n; m) ! n + m y se
cumple que:

1. 0 + m = m
2. sig(n) + m = sig (n + m)

Tomemos por ejemplo: n = 1 y m = 3 e ilustremos las dos condiciones


anteriores.

0+3 = 3
sig (1) + 3 = 2 + 3 = 5 = sig (1 + 3) = sig (4)

1.3.2 Denicin. Denimos la multiplicacin de nmeros naturales como una


aplicacin : N N ! N; de forma tal que (n; m) ! n m y se cumple
que:

1. 0 m = 0
2. m sig(n) = m n + m

Tomemos por ejemplo: n = 2 y m = 3 e ilustremos las dos condiciones.

0 3 = 0
3 sig (2) = 3 2 + 3 = 6 + 3 = 9

1.3.1 Teorema.

Se verican las propiedades asociativa, conmutativa y cancelativa para la


suma de nmeros naturales:

1. Propiedad asociativa: 8a; b; c 2 N; (a + b) + c = a + (b + c)

14
2. Propiedad conmutativa: 8a; b 2 N; a + b = b + a
3. Propiedad cancelativa: 8a; b; c 2 N; a + c = b + c =) a = b

Demostracin:

1. Propiedad Asociativa. 8a; b; c 2 N; (a + b) + c = a + (b + c)

Haremos induccin sobre a:

Si a = 0; entonces

(0 + b) + c = b + c = 0 + (b + c)

podemos asumir, la hiptesis de induccin

(a + b) + c = a + (b + c)

Ahora debemos probar para sig(a) = a + 1, esto es

[sig (a) + b] + c = sig (a) + (b + c)

[sig (a) + b] + c =) sig (a + b) + c def. de sig


=) sig [(a + b) + c] def. de sig
=) sig [a + (b + c)] hip. inductiva
=) sig (a) + (b + c) def. de sig

2. Propiedad Conmutativa: 8a; b 2 N; a + b = b + a

Haremos induccin sobre a:


Si a = 0, entonces
0+b=b=0+b
podemos asumir, la hiptesis de induccin

a+b=b+a

Ahora debemos probar para sig(a) = a + 1, esto es

sig (a) + b = b + sig (a)

sig (a) + b =) sig(a + b) def. sig


=) sig (b + a) hip. inductiva
=) (b + a) + 1 def. sig
=) b + (a + 1) prop. asociativa
=) b + sig(a) def. sig

15
3. Propiedad Cancelativa: 8a; b; c 2 N; a + c = b + c =) a = b

Haremos esta demostracin haciendo induccin sobre c:


Si c = 0; entonces
a + 0 = b + 0 =) a = b
Asumimos la hiptesis inductiva

a + c = b + c =) a = b

Ahora debemos probar para sig(c) = c + 1; esto es

a + (c + 1) = b + (c + 1) =) a = b

a + (c + 1) = b + (c + 1) =) (a + c) + 1 = (b + c) + 1 prop. asociativa
=) sig (a + c) = sig (b + c) def. siguiente
=) a+c=b+c inyectividad de la aplicacin sig
=) a=c hiptesis inductiva

1.3.2 Teorema.

Se verican las propiedades asociativa, conmutativa, cancelativa y distribu-


tiva (respecto a la suma) para el producto de nmeros naturales:

1. Propiedad asociativa: 8a; b; c 2 N; (ab)c = a(bc)


2. Propiedad conmutativa: 8a; b 2 N; ab = ba
3. Propiedad cancelativa: 8a; b; c 2 N; ac = bc =) a = b
4. Propiedad didtributiva: 8a; b; c 2 N; a (b + c) = ab + ac

1.4 Exponenciacin en N

1.4.1 Denicin. Para cualesquiera nmeros naturales a y n se tiene que:


i.

an : =a
| a a{z a a}
n veces

ii.
a0 = 1

16
iii.

asig(n) = an a

1.4.1 Teorema. 8a; b 2 N y para cualquier m; n 2 N :

1.
am an = am+n

2.
n
(am ) = am n

3.
n
(ab) = an bn

Demostracin:

1. Vamos a probar la primera parte del teorema 1.4.1, esto es: am an = am+n
i) Para n = 0; la propiedad es vlida puesto que: am+0 = am = am 1 = am a0 :
ii) Ahora vamos a suponer que existen nmeros a; m; n tales que no cumplen
la propiedad, es decir: am an 6= am+n

Por lo dicho en i) n 6= 0 y por lo tanto debe existir n1 de forma que n1 +1 = n:


Se sigue entonces que

n > n1 y am an1 6= am+n1


Esto ltimo porque n1 < n1 + 1 = n y si am an1 = am+n1 ; entonces am an =
m n1 +1
a a = (am an1 ) a1 = (am+n1 ) a1 = am+(n1 +1) = am+n ; que contradice
nuestro supuesto inicial.
Aplicando el mismo razonamiento a n1 encontramos otro nmero natural n2
tal que

n > n1 > n2 y am an1 6= am+n1


Continuamos de este modo para cada ni ; encontramos que existe una suce-
sin innita decreciente n > n1 > n2 > n3 > ::: > ni > ::: de nmeros naturales.
Lo cual es imposible puesto que cualquier subconjunto de nmeros naturales
tiene elemento mnimo (PBO). Por lo tanto, am an = am+n para cualesquiera
nmeros naturales m; n y a:

17
n
2. Continuaremos con la segunda parte del teorema: (am ) = am n
0
Particularmente, n = 0 tenemos la clara igualdad; (am ) = 1 = a0 = am 0
1
en el caso n = 1; tambin vericamos que; (am ) = am = am 1
As, asumimos que esto es cierto para algn nmero natural k; nuestra
k
hiptesis inductiva es entonces; (am ) = am k
k+1
debemos probar que es cierta para sig (k) = k + 1; i.e, (am ) = am(k+1)
Procedemos como sigue:

k+1 k
(am ) = a
|
m
am am
{z a
m
am} = (am am am am ) am = (am ) am = amk am
| {z }
(k + 1) veces k veces

esto por denicin de potencia e hiptesis inductiva. Luego por la parte 1)


del teorema, tenemos lo siguiente:

amk am = akm+m = am(k+1)


n
que es lo que queriamos demostrar, por tanto (am ) = am n es vlido,
8a; m; n 2 N:

La tercera parte, para completar el teorema, queda como ejercicio.

18
1.5 Ejercicios resueltos sobre axiomas de Peano e Induc-
cin.

1. Denimos la relacin "menor o igual que" ( ) de modo siguiente: 8a; b 2


N; a b () 9x 2 N : a + x = b:
Probar que ( ) es relacin de orden, es decir, es reexiva, antisimtrica y
transitiva.

Demostracin.

i. Es reexiva: 8a 2 N; 9 0 2 N : a + 0 = 0 + a = a =) a a
ii. Antisimtrica: 8a; b 2 N; a b^b a =) a = b

Si a b =) 9 x 2 N : a + x = b; luego, Si b a =) 9 y 2 N : b + y = a;
Escribamos ahora: a + x = b = b + (y + x) =) y = x = 0 y de aqu a = b

iii. Transitiva: Si a b^b c =) a c:

Si a b =) 9 x 2 N : a + x = b ^ si b c =) 9 y 2 N : b + y = c; entonces
c = b + y = (a + x) + y = a + (x + y) ; es decir existe el nmero (x + y) = z 2 N;
tal que c = a + z =) a c:

2. Ningn nmero natural coincide con su siguiente: 8n 2 N; n 6= sig (n)

Demostracin.
Sea A = fn 2 N : n 6= sig (n)g : Veamos que tal conjunto coincide con N:

0 2 A; puesto que por axioma 3 0 6= sig (0)


8n 2 A; n 6= sig(n) =) sig(n) 6= sig(sig(n)) , por axioma 4 (inyectividad
de la funcin siguiente). Luego sig(n) 2 A

Y del axioma quinto, se sigue que A = N:

3. Todo nmero natural es estrictamente menor que su siguiente: 8a 2 N; a <


sig (a)

Demostracin.
sig (a) = sig (0 + a) = sig (0)+a =) 9 sig (0) 2 N : a+sig (0) = sig (a) =)
a sig (a) ; por el ejercicio anterior tenemos que n 6= sig(n); por tanto:
a sig (a) ^ a 6= sig(a) =) a < sig (a)

19
4. Demustrese en N;
a 6= b =) a + n 6= b + n

Hacemos induccin sobre n y tenemos:

a. n = 1 =) a + 1 = sig (a) 6= a 6= b 6= sig (b) = b + 1; de aqu que a + 1 6= b + 1:


Luego nuestra hiptesis de induccin a 6= b =) a + h 6= b + h; 8h 2 N
b. a + h 6= b + h =) a + sig (h) 6= b + sig (h) : Procedemos como sigue: a + h 6=
b + h =) sig (a + h) 6= sig (b + h) =) a + sig (h) 6= b + sig (h) :

Demostraciones por induccin.

1. Prubese la validez de la siguiente suma:


1 1 1 1 n
Sn = + + + ::: + =
1 2 2 3 3 4 n (n + 1) n+1

Casos particulares n = 1; n = 2
1
S1 = (evidente)
2

1 1 1 1 3+1 4 2 n 2 2
S2 = + = + = = = por otro lado tenemos; = =
1 2 2 3 2 6 6 6 3 n+1 2+1 3

Hiptesis inductiva, para n = k,


1 1 1 1 k
Sk = + + + + =
1 2 2 3 3 4 k (k + 1) k+1
donde k es un nmero natural.

Demostremos que, tambin es vlida para n = k + 1; i:e


k+1
Sk+1 =
k+2
En efecto,

1 1 1 1 1
Sk+1 = + + + + +
1 2 2 3 3 4 k (k + 1) (k + 1) (k + 2)

por consiguiente podemos escribir


1
Sk+1 = Sk +
(k + 1) (k + 2)

20
que por hiptesis de induccin reescribimos como

k 1 k 2 + 2k + 1 k+1
Sk+1 = + = =
k + 1 (k + 1) (k + 2) (k + 1) (k + 2) k+2
hemos demostrados ambas condiciones, ahora en virtud del PIM podemos
armar que la proposicin anterior es verdadera.

2. Prubese que todo nmero natural impar es de la forma

Pn = 2n 1

Caso particular n = 1; n = 2

P1 = 2 (1) 1=1

P2 = 2 (2) 1=3
lo que verica la primera parte de la induccin.

Hiptesis inductiva, para n = k,

Pk = 2k 1

Demostremos, entonces que, la frmula debe ser vlida para (k + 1) ; esto es


Pk+1 = 2 (k + 1) 1 es impar.

Pk+1 = 2 (k + 1) 1 = 2k + 1

Para obtener el (k + 1)-simo nmero impar basta agregar 2 al k esimo


nmero impar:

Pk+1 = Pk + 2
pero, por hiptesis, Pk = 2k 1 de modo que

Pk+1 = (2k 1) + 2 = 2k + 1
como queramos demostrar.

3. Calclese la suma de los n primeros nmeros impares.

21
Llamemos Sn a la suma buscada:

Sn = 1 + 3 + 5 + + (2n 1)
Tomenos sucesivos valores para n; hasta obtener informacin suciente para
poder enunciar una hiptesis acertada, para posteriormente demostrarla por
induccin.

S1 = 1; S2 = 1 + 3 = 4; S3 = 1 + 3 + 5 = 9; S4 = 1 + 3 + 5 + 7 = 16;
S5 = 1 + 3 + 5 + 7 + 9 = 25; S6 = 1 + 3 + 5 + 7 + 9 + 11 = 36;
S7 = 1 + 3 + 5 + 7 + 9 + 11 + 13 = 49

Resulta de manera casi natural que

S1 = 12 ; S2 = 22 = 4; S3 = 32 = 9; S4 = 42 = 16;
S5 = 52 = 25; S6 = 62 = 36 y S7 = 72 = 49

Sobre esta base podemos suponer que

Sn = n 2

Demostremos que esta hiptesis es verdadera


De los clculos anteriores resulta que la hiptesis es vlida para n = 1:

Hiptesis inductiva, para n = k,


2
Sk+1 = (k + 1)

Probaremos que tambin debe ser vlida para n = k + 1;

En efecto,
Sk+1 = Sk + (2k + 1)
2
pero Sk = k ; de modo que
2
Sk+1 = k 2 + (2k + 1) = (k + 1)
como queriamos demostrar.

22
4. Demustrese la desigualdad de Bernoulli
n
(1 + ) > 1 + n

donde > 0 y n es un nmero natural mayor que 1.

Demostracin.
La desigualdad es vlida para n = 2 puesto que:

2 2 2
(1 + ) = 1+2 + y > 0 de aqu se deduce que
2
(1 + ) > 1+2

Hiptesis inductiva, para n = k;


k
(1 + ) > 1 + k ( )
Demostremos entonces que la desigualdad tambin se cumple para n = k +1;
o sea, que
k+1
(1 + ) > 1 + (k + 1)
En efecto, por hiptesis, se tiene

1+ >0

de modo que es vlida la desigualdad


k+1
(1 + ) > (1 + k ) (1 + )
que se obtiene multiplicando por (1 + ) ambos miembros de la igualdad ( )
luego reescribiendo la ltima desigualdad tenemos
k+1 2
(1 + ) > 1 + (k + 1) +k
2
descartando el sumando k de la derecha de la ltima desigualdad, obten-
emos
k+1
(1 + ) > 1 + (k + 1)
que es lo queramos probar.

23
1.6 Misterios de los Nmeros Naturales.

Observe la siguiente relacin numrica


100 = 13 + 23 + 33 + 43
Cien como la suma de los primero cuatro naturales elevados al cubo. Otra
relacin interesante es la del nmero de das del ao, es decir, 365

102 + 112 + 122 = 365


Resulta ser la suma de los cuadrados de tres nmeros consecutivos em-
pezando con 10:

Pero tambin es
132 + 142 = 365
la suma de los cuadrados de los siguentes nmeros.

En otra forma
102 + 112 + 122 + 132 + 142
= 365
2

Ahora, observe si elevamos a la quinta potencia todas las cifras 54748 y


sumamos el resultado

55 + 45 + 75 + 45 + 85 = 54 748
Y, no menos curiosa la siguiente relacin
13 + 23 + 33 + 43 + 53 + 63 + 53 + 43 + 33 + 23 + 13 = 666

1.7 Ejercicios Propuestos Sobre Nmeros Enteros

1. Prubese que, si a > b; entonces ac > bc


2. Demustre el principio de buen orden (PBO)
3. Demustre el teorema 1.4.1 parte 3
4. Demustrese que la suma de los cuadrados de los n primeros nmeros
naturales es igual a:
n (n + 1) (2n + 1)
12 + 22 + 32 + + n2 = ; i:e
6
n
X n (n + 1) (2n + 1)
i2 =
i=1
6

24
5. El producto 1 2 3 n se indica por n! (se lee ene factorial) : Donde
1! = 1; 2! = 2; 3! = 6; 4! = 24: Calclese

Sn = 1 1! + 2 2! + 3 3! + + n n!

y prubese por induccin.

6. Para qu valores naturales de n se cumple la desigualdad 2n > 2n + 1?


Pruebe la desigualdad a partir del n encontrado
7. Demestrese que
1 1 1 p
p
2
+ p
2
+ + p > 2n
1 2 2
n
para todo natural n > 1:
8. Demustrese el teorema: La media geomtrica de varios nmeros naturales
no pasa de la media aritmtica de los mismos, es decir, siendo a1 ; a2 ; : : : ; an
unos nmero naturales, se tiene
p a1 + a2 + : : : + an
n
a1 ; a2 ; : : : ; an
n

9. Demustrese: n planos que pasan por un mismo punto, sin que contengan
nunca tres una recta comn, dividen el espacio en

An = n (n 1) + 2

partes.
10. Demustrese que
n n n n
(1 + i) = 2 2 cos + i sin
4 4
p
donde i = 1:
11. Demustrese el Binomio de Newton.
n
X
n n n k k
(a + b) = a b
k
k=0

25
2 Nmeros Enteros y Los nmeros Primos

"Dios puede que no juega a los dados con el universo, pero algo extrao est
pasando con los nmeros primos"
Paul Erds.

Desarrollaremos aqu las ideas ms fundamentales de la aritmtica: la di-


visibilidad, el mximo comn divisor, mnimo comn mltiplo y la denicin
de nmero primo, entre otros. Adems de tratar el teorema fundamental de la
aritmtica y algunas de sus aplicaciones en la matemtica.

2.1 Divisibilidad de Nmeros Enteros


El conjunto
Z=f 3; 2; 1; 0; 1; 2; 3 g
es llamado el conjunto de nmeros enteros; los nmeros

Z+ [ 0 = f0; 1; 2; 3; g

enteros no negativos; y los nmeros

Z+ = f1; 2; 3; g

enteros positivos.

Usualmente denotamos a los nmeros enteros con letras latinas mnusculas

a; b; ;n ; x; y; z

algunas veces para referirnos a una subclase de los enteros usamos la misma
notacin, as que, basta con indicar a cual hacemos referencia. (positivos, neg-
ativos, etc.)

2.1.1 Denicin de divisibilidad. Diremos que d divide a n y escribiremos


djn; si n = cd para un c 2 Z: Diremos tambin que n es un mltiplo
de d; que d es un divisor de n; o que d es un factor de n: Si d no
divide a n escribiremos d - n:

2.1.1 Teorema. La divisibilidad verica las siguientes propiedades:


a. njn (propiedad reexiva)
b. djn ^ njm; entonces djm (propiedad transitiva)

26
c. djn ^ djm; entonces dj (an + bm) (propiedad lineal)
d. djn () adjan y a 6= 0 (propiedad de multiplicacin)
e. 1jn (1 divide a todos los enteros)
f. djn ^ djm =) djn + m
g. djn + m ^ djn =) djm

Demostracin.

a. njn (propiedad reexiva)

n=n =) reexividad de la igualdad


=) n = 1 n identidad multiplicativa en Z
=) njn por denicin de divisibilidad

b. djn ^ njm; entonces djm (propiedad transitiva)

djn =) n = d c def. divisibilidad


njm =) m = n c; def. divisibilidad
m = (d c) c; sustituyendo n = d c
m = d (c c; ) Prop. asociativa de la multiplicacin
m = d c;; es ley de composicin interna
) djm def. divisibilidad

c. djn ^ djm; entonces dj (an + bm) (propiedad lineal)

djn =) n = d c def. divisibilidad


djm =) m = d c; def. divisibilidad
an = d c a multiplicando por a ambos lados
bm = d c; b multiplicando por b ambos lados
an + bm = d (c a) + d (c; b) Sumando miembro a miembro
an + bm = d x+d y + y son leyes de composicin interna
an + bm = d (x + y) Prop. distributiva
) dj (an + bm) def. divisibilidad

d. djn () adjan y a 6= 0 (propiedad de multiplicacin)

djn () n = d c def. divisibilidad


() an = (ad) c multiplicado ambos lados por a
() adjan def. divisibilidad

e. 1jn, es evidente, puesto que n = 1 n

27
f. djn ^ djm =) djn + m

djn =) n = d c def. divisibilidad


djm =) m = d c0 def. divisibilidad
=) n+m = d (c + c0 ) sumando ambas igualdades
=) n+m = dz c + c0 = z
=) djn + m def. divisibilidad

g. djn + m ^ djn =) djm

djn + m =) n+m = d c def. divisibilidad


djn =) n = d c00 def. divisibilidad
=) (n + m) n = d (c c00 ) restando ambas igualdades
=) m = dz 0 c + c00 = z 0
=) djm def. divisibilidad

2.2 Mximo Comn Divisor

2.2.1 Teorema. El algoritmo de la divisin. Dados dos enteros cua-


lesquiera n y m; con m > 0; existen los enteros nicos q y r tales que
n = mq + r; 0 r < m

Demostracin.
Sea A el conjunto de enteros no negativos dado por

A = fx : x = n my; y 2 Z; x 0g
Es un conjunto no vaco de enteros no negativos, en virtud del PBO admite
mnimo, que designaremos n mq: Entonces n = mq + r y r 0: Ahora
demostraremos que r < m: Supongamos r m: Entonces 0 r m < r: Pero
r m 2 A ya que r m = n m(q + 1): Por lo tanto r m es un elemento de
A menor que su elemento mnimo r. Esta contradiccin demuestra que r < m:

Los nmeros q; r son nicos, ya que si existiesen otros con estas condiciones
q ; ; r; ; entonces mq + r = mq ; + r; ; de donde m jq q ; j = jr; rj : Luego si
jq q ; j > 0; entonces jq ; qj 1; por lo que jr; rj = m jq q ; j m ( ):
Ahora tenemos que r 0; implica r 0 y as r; r r; < m luego
r < m m + r; =) m < r; r: Entonces m < r; r < m; es decir
;
jr rj < m; lo que constituye una contradiccin para ( ) ; lo cual completa la
demostracin.

28
2.2.1 Denicin. Un nmero entero positivo d se denomina divisor comn
de los nmeros enteros positivos n1 ; n2 ; n3 ; ; ni si y solo si d es
divisor de n1 ; n2 ; n3 ; ; ni : Puesto que solamente existe un nmero
nito de divisores de cualquier entero diferente de cero, solamente
existen un nmero nito de divisores comunes de n1 ; n2 ; n3 ; ; ni ;
luego el mayor de los divisores comunes se llama mximo comn di-
visor (mcd) de n1 ; n2 ; n3 ; ; ni ; y se denota mcd (n1 ; n2 ; n3 ; ; ni )
o simplemente (n1 ; n2 ; n3 ; ; ni ) : Si el mximo comn divisor es 1
decimos que los nmeros son coprimos.

2.2.2 Teorema. Algoritmo de Euclides. Se dan dos enteros positivos


n y m, tales que m - q. Se escribre r0 = n; r1 = m; y aplicando
repetidamente el algoritmo de la divisin obteniendo un conjunto de
restos r2 ; r3; ; rn ; rn+1 denidos sucesivamente por las relaciones

r0 = r1 q 1 + r2 0 < r2 < r1 ;
r1 = r2 q 2 + r3 0 < r3 < r2 ;
.. ..
. .
rn 2 = rn 1 q n 1 + rn 0 < rn < rn 1;
rn 1 = rn qn + rn+1 rn+1 = 0

Entonces rn ; es el ltimo resto no nulo de este proceso, es mcd (n; m) ; el


mximo comn divisor de n y m:

Demostracin.
Existe un momento en que rn+1 = 0 puesto que los ri son decreciente y no
negativos. La ltima relacin, rn 1 = rn qn demuestra que rn jrn 1 : La anterior
a la ltima prueba que rn jrn 2 : Por induccin vemos que rn divide a cada ri :
En particular rn jr1 = m y rn jr0 = n; luego rn es un divisor comn de n y m:
Ahora sea d otro divisor comn de n y m: La denicin de r2 prueba que djr2 :
La relacin que le sigue prueba que djr3 : Por induccin, d divide a cada ri luego
djrn : Por lo tanto rn es el mcd requerido.

2.2.3 Teorema. Si d es el mximo comn divisor de n y m, entonces existen


los enteros a y b tales que d = mcd (n; m) = an + bm

Demostracin.
Partiendo del teorema 2.2.2 tenemos
n = mq1 + r2 0 < r2 < r1 ;
m = r2 q 2 + r 3 0 < r3 < r2 ;
.. ..
. .
rn 2 = rn 1 q n 1 + rn 0 < rn < rn 1;
rn 1 = rn qn + rn+1 rn+1 = 0

29
Al considerar las ecuaciones en el orden propuesto, se obtiene r2 como com-
binacin lineal de n y m :
r2 = n mq1
sustituyendo en la segunda ecuacin se obtiene r3 como combinacin lineal
de n y m :

r3 = m r2 q 2
r3 = m q2 (n mq1 )

Si reiteramos este procedimiento, entonces en la penltima ecuacin obten-


emos rn ; que es el mcd de n y m; como combinacin lineal de estos nmeros.

2.2.4 Teorema. El mcd posee las siguientes propiedades:


a. (n; m) = (m; n)
b. (n; (m; s)) = ((n; m) ; s)
c. (cn; cm) = jcj (n; m)
d. (n; 1) = (1; n) = 1
n m (n;m)
e. Si djn y djm y d > 0; entonces d; d = d : A dems, si (n; m) = g;
n m
entonces g; g = 1:

Demostracin.

a. Si d = (n; m), es claro que (n; m) = (m; n) : puesto que independientemente


del orden d dividir a n, m y podr expresarse como combinacin lineal
de ellos.
b. Si d = (n; (m; s)) ; llamemos d0 = (m; s) ; por el teorema 2.2.3 se tiene que
d0 = mx + sy luego d = an + (mx + sy) b: De aqu se sigue que d =
[an + m (xb)] + s (yb) ; eligiendo a = kx; se tiene d = [n (kx) + m (xb)] +
s (yb) = [nk + mb] x+s (yb) : Finalmente tomando d1 = (n; m) = nk +mb;
se obtiene d = d1 (x) + s (yb) ; por lo tanto d = (d1 ; s) = ((n; m) ; s)
c. Sea d = (n; m) y sea e = (cn; cm) : Queremos demostrar que e = jcj d:
Escribimos d = nx + my; por el teorema 2.2.3. Entonces tenemos cd =
ncx + mcy ( ) : Por lo tanto cdje puesto que cd divide a nc y mc: Adems,
la ecuacin ( ) prueba que ejcd puesto que ejnc y ejmc: Por lo tanto
jej = jcdj ; e = jcj d

30
2.2.5 Teorema. Lema de Euclides. Si njmc y si (n; m) = 1; entonces njc

Demostracin. Puesto que (n; m) = 1 podemos escribir 1 = na + mb (por


el teorema 2.2.3). Por consiguiente c = nac + mbc: Pero njnac y njmbc; luego
njc (por teorema 2.1.1.f)

2.2.1 Ecuaciones Lineales Diofnticas.

Una ecuacin lineal diofntica es una ecuacin de la forma

na + mb = c
donde n; m y c son nmeros enteros y a; b variables que recorren todo Z:

2.2.6 Teorema. Si (n; m) jc entonces la ecuacin lineal diofntica na + mb = c


siempre tiene solucin entera.

Demostracin. Supongamos que la ecuacin na + mb = c tiene como


solucin a1 y b1 y que d = (n; m) no divide a c y sean a0 y b0 una solucin para
na + mb = c:

De esta manera tenemos

na1 + mb1 = c
na0 + mb0 = d

Por el algoritmo de la divisin tenemos c = qd + r con 0 < r < d puesto que


d no divide a c:

Entonces
na1 + mb1 = qd + r
= q (na0 + mb0 ) + r
= n (qa0 ) + m (qb0 ) + r
y por tanto

n (a1 qa0 ) + m (b1 qb0 ) = r


Lo anterior es una contradiccin, puesto que hemos obtenido una combi-
nacin lineal de n y m y 0 < r < d; esto contradice que d era la combinacin
lineal mnima (es decir, d no era el mximo comn divisor). La contradiccin
suirgi de suponer que na + mb = c poda tener solucin aunque d - c; por lo
que hemos demostrado el teorema.

31
2.2.7 Teorema. La ecuacin na + mb = c tiene solucin si y slo si d = (n; m)
divide a c: Adems, las soluciones son los nmeros de la forma.

mt nt
a = a0 + ; b = b0
d d
donde a0 y b0 son soluciones particulares y t es cualquier entero.

Ejemplo. Encontrar los valores x y y que satisfaga

243x + 198y = 9 ( )
Primeramente vericamos que tenga solucin, esto es que (243; 198) divida
a 9: Como (243; 198) = 9 ( ) tiene solucin.

243x + 198y = 9 dividiendo por 9 obtenemos


27x + 22y = 1 ( ) que es equivalente a ( )

Una solucin particular para ( ) es x = 9 y y = 11; puesto que, 27 (9) +


22 ( 11) = 1: As la soluciones para ( ) son

x = 9 + 22t; y= 11 27t
En particular para t = 1 tenemos; x = 31; y = 38 que verican 243 (31) +
198 ( 38) = 9:

2.3 Mnimo Comn Mtiplo

2.3.1 Denicin. Los nmeros enteros positivos n1 ; n2 ; n3 ; ; ni ; tienen un


mltiplo comn b si nj jb para j = 1; 2; 3; : : : ; i: (Ntese que existen
mltiplos comunes; por ejemplo, el producto n1 n2 n3 ni )
El menor de los mltiplos comunes positivos recibe el nombre de
mnimo comn mltiplo y se denota por [n1 ; n2 ; n3 ; ; ni ] :

2.3.1 Proposicin: El mnimo comn multiplo cumple con las siguientes


propiedades.
a. njm; si [n; m] = jmj
b. c > 0 implica [cn; cm] = c [n; m]
n m [n;m]
c. d > 0; djn y djm implica d; d = d

32
2.3.1 Teorema. Si b es cualquier mltiplo comn de n1 ; n2 ; n3 ; ; ni ; en-
tonces [n1 ; n2 ; n3 ; ; ni ] jb: Esto equivale a decir que si h denota a
[n1 ; n2 ; n3 ; ; ni ] ; entonces 0; h; 2h 3h; ::: incluyen todos los
mltiplos comunes de n1 ; n2 ; n3 ; ; ni :

Demostracin. Sea b cualquier mltiplo comn, divdase b entre h. Por el


teorema 2.2.1, existen un cociente q y un residuo r, tales que, b = qh + r; 0
r < h: Debe probarse que r = 0: Si r 6= 0 se argumenta del modo siguiente.
Para cada i = 1; 2; :::; j se sabe que ni jh y ni jb; de modo que ni jr: As que, r es
un mltiplo comn positivo de n1 ; n2 ; n3 ; ; ni contrario al hecho de que h es
el menor positivo de todos los mltiplos comunes. Por tanto debe ser r = 0 y
hjb:

2.3.2 Teorema. Si c > 0, [cn; cm] = c [n; m] : Tambin [n; m] (n; m) = jn mj

Demostracin. Recordemos que, si ajb ^ bja =) a = b: Ya que [cn; cm] es


un mltiplo de cn; con mayor razn es mltiplo de c y por tanto, puede escribirse
en la forma ch1 : Denotando [n; m] por h2 ; se observa que njh2 ; mjh2 ; cnjch2 y
cmjch2 ;entonces, ch1 jch2 : De donde h1 jh2 : Por otra parte, cnjch1 ; cmjch1 ; njh1 ; mjh1
y as h2 jh1 : Se concluye que h1 = h2 y as se establece la primera parte del teo-
rema.

Empecemos con el caso especial donde (n; m) = 1: Ahora bien, [n; m] es


un mltiplo de n, digamos cn: Entonces mjcn y (n; m) = 1; as por el teorema
2.2.5, mjn: De aqu que m c; mn cn: Pero nm; siendo un mltiplo comn
positivo de n y m, no puede ser menor que el mnimo comn mltiplo y, por
tanto, nm = mn = [n; m] :
Regresemos al caso general, donde (n; m) = g > 1; se tiene ng ; m
g = 1: Al
aplicar el resultado del prrafo precedente, se obtiene

n m n m n m
; ; =
g g g g g g
Al multiplicarse por g 2 y usando el teorema 2.2.4.e y la proposicin 2.3.1,
as como la primera parte del presente teorema, se obtiene [n; m] (n; m) = nm

2.4 Nmeros Primos

2.4.1 Denicin. Un entero n se llama primo si n > 1 y si los nicos divisores


positivos de n son 1 y n: Si n > 1 y no es primo, entonces n se llama
compuesto.
Ejemplos: Los nmeros primos menores que 100 son:

33
2; 3; 5; 7; 11; 13; 17; 19; 23; 29; 31; 37; 41; 43; 47; 53; 59; 61; 67; 71; 73; 79; 83; 89 y 97.

2.4.1 Teorema. Cada entero n > 1 es primo o es producto de primos.

Demostracin. Sea n > 1: Supongamos que no es divisible por ningn


primo. En particular, n mismo no puede ser primo pues sera divisible por si
mismo. Como n no es primo, tiene algn divisor positivo d1 distinto de 1 y n;
es decir 1 < d1 < n. Pero d1 no puede ser primo por que dividira a n: Entonces
existe un d2 que divide a d1 tal que 1 < d2 < d1 < n: Como d2 no puede ser
primo porque divide a n; repetimos el argumento con d2 para obtener d3 tal que
1 < d3 < d2 < d1 < n: Como d3 no puede ser primo existe un d4 que divide a d3
y as sucesivamente. Esto lleva a una contradiccin pues no es posible continuar
indenidamente este proceso ya que entre 1 y n slo hay un nmero nito de
trminos. Por tanto, n debe ser divisible entre algn primo.

2.4.2 Teorema. (Euclides)Existen innitos nmeros primos.

Demostracin. Sea n un nmero natural arbitrario. Sabemos que n y


n + 1 son nmeros enteros positivos consecutivos, deben ser coprimos. Entonces
el nmero N2 = n(n+1) debe tener, como mnimo, dos factores primos distintos.
Anlogamente, los nmeros enteros n(n + 1) y n(n + 1) + 1; son consecutivos y,
por tanto, coprimos . En consecuencia, el nmero N3 = n(n + 1) [n(n + 1) + 1]
debe tener, como mnimo, tres factores primos diferentes. Este proceso puede
ser continuado indenidamente, as que el conjunto de los nmeros primos es
innito.

2.4.3 Si un primo p no divide a n; entonces (p; n) = 1

Demostracin. Sea d = (p; n) : Entonces djp; como p es primo se tiene que


d = 1 d = p: Pero djn luego por hiptesis debe ser d 6= p: En consecuencia
d = 1:

2.4.4 Teorema. Si un primo p divide a nm; entonces pjn pjm: En general,


si un primo p divide a un producto n1 n2 n3 ni ; entonces p divide a uno,
por lo menos, de los factores.

Demostracin. Supongamos que pjnm y p - n: Veremos que pjm: Por el


teorema 2.4.3, (p; n) = 1; luego por el lema de Euclides (teorema 2.2.5), pjm:
La generalizacin para el producto n1 n2 n3 ni queda a modo de ejercicio.

34
2.4.5 Teorema. Existen arbitrariamente grandes vacos en la serie de los
primos. Dicho de otra manera, dado cualquier entero k; existen k enteros
compuestos consecutivos.

Demostracin. Recordemos el factorial de un nmero. Sea k un entero el


factorial de k k! es: k! = 1 2 3 k
Considrense los enteros

(k + 1)! + 2; (k + 1)! + 3; ; (k + 1)! + k; (k + 1)! + k + 1:


Cada uno de estos es compuesto porque j divide a (k + 1)!+j si 2 j k+1:

Los nmeros primos estn espaciados irregularmente, tal y como lo sugiere


el ltimo teorema. Si denotamos el nmero de primos que no excede a x por
(x) ; podra preguntarse acerca de la naturaleza de esta funcin. Uno de los
resultados ms impresionantes de la teora avanzada de nmeros, es

(x) log x
lim =1
x !1 x
Este resultado notable se llama el teorema del nmero primo, y su
demostracin se la debemos a J. Hadamard y C.J de la Valle Poussin quienes
independientemente lo demostraron en 1896.

2.5 El teorema Fundamental de la Aritmtica.

2.5.1 Teorema fundamental de la aritmtica. Cada entero n > 1 se puede


representar como producto de factores primos en forma nica, salvo el
orden de los factores.

Demostracin. Usaremos la induccin sobre n: El teorema es verdadero


para n = 2: Suponemos, entonces, que es verdadero para todo entero mayor que
1 y menor que n: Probaremos que es verdadero tambin para n: Si n es primo
no hay nada que probar. Por lo tanto suponemos que n es compuesto y que
admite dos descomposiciones, que son

n = p1 p2 pi = q 1 q 2 qj ( )
Queremos demostrar que i = j y que cada p es igual a algn q: Dado que p1
divide al producto q1 q2 qj debe dividir a uno, por lo menos, de los factores.
Ordenaremos los q1 q2 qj de forma que p1 jq1 : Entonces p1 = q1 ya que p1 y q1
son primos. En ( ) podemos dividir por p1 obteniendo
n
= p2 pi = q 2 qj
p1

35
Si i > 1 j > 1; entonces 1 < pn1 < n: La hiptesis de induccin nos dice
que las dos descomposiciones de pn1 son idnticas, prescindiendo del orden de los
factores. Por consiguiente i = j las descomposiciones de ( ) son tambin idn-
ticas, si prescindimos del orden de los factores. Esto completa la demostracin.

En la descomposicin de un nmero n; un cierto primo p puede aparecer ms


de una vez. Si los factores primos distintos de n son p1; p2 pr y si pi aparece i
veces como factor, escribiremos

n = p1 1 pj j

o ms compactamente
j
Y
n= pi i
i=1

Si la descomposicin en factores primos de n es p1 1 pj j , entonces todos


los divisores de n son los nmeros de la forma

d = p 1 1 p2 2 pj j ; 0 1 1; 0 2 2; : : : ; 0 j j

Por ejemplo.

15925 = 52 72 13
1800 = 23 32 52

2.5.1 Denicin. Por [x] denotamos la funcin parte entera de x. Se dene


para todos los valores reales de x y representa el entero mayor, no
superior a x:

[x] : R ! Z

Ejemplos.

[5] = 5; [2:3] = 2; [ 3:25] = 4

2.5.2 Teorema. El exponente, con el que un nmero primo p gura en el


producto n!; es igual a

n n n n
+ 2 + 3 + +
p p p pi

36
Demostracin. En efecto,
h i el nmero de factores en el producto
h i n! que son
mltiplos de p; es igual a p ; entre ellos, mltiplos de p hay pn2 ; entre estos
n 2
h i
ltimos, mltiplos de p3 hay pn3 ; etc. La suma de los nmeros indicados da
precisamente el exponente buscado, puesto que cada factor en el producto n!
que sea mltiplo de pm ; pero no de pm+1 ; se cuenta del modo indicado m veces,
como mltiplo de p; p2 ; p3 ; : : : y, nalmente, de pm :

Ejemplo. El exponente con el que el nmero 5 gura en el producto 35! es


igual a

35 35
+ =7+1=8
5 25

2.6 Ejercicios Resueltos sobre divisibilidad y factores pri-


mos.

1. Probar que cualquiera que sea n 2 N 11j32n+2 + 26n+1

Solucin. Procederemos haciendo induccin en n:


Para n = 1; 32(1)+2 + 26(1)+1 = 209 = 19 11
2k+2
Hiptesis inductiva 11j3 +26k+1
2(k+1)+2
Tsis de induccin 11j3 +26(k+1)+1

32(k+1)+2 +26(k+1)+1
32k+2 32 +26k+1 26 reescribiendo la tsis
32k+2 32 +26k+1 26 +0 elemento neutro de (Z; +)
32k+2 32 +26k+1 26 +32 26k+1 32 26k+1 0 = 32 26k+1 32 26k+1
32k+2 32 +32 26k+1 + 26k+1 26 32 26k+1 Asociatividad
32 32k+2 + 26k+1 +26k+1 26 32 Prop. distributiva
2
32 32k+2 +26k+1 +26k+1 55 11j3 32k+2 + 26k+1 hiptesis y 55 = 11 5
) 11j3
2(k+1)+2 2 6k+1
+26(k+1)+1 11j3 32k+2 + 26k+1 ^11j2 55
luego divide a suma

2. Probar que 34n+2 +2 43n+1 es mltiplo de 17

Solucin. Procederemos haciendo induccin en n:


Para n = 1; 34(1)+2 + 2 43(1)+1 = 1241 = 73 17 Mltiplo de 17
Hiptesis inductiva 17j 34k+2 + 2 43k+1
Tesis de induccin 17j 34(k+1)+2 + 2 43(k+1)+1

37
34(k+1)+2 +2 43(k+1)+1 = 34k+2 34 +2 43k+1 43 (reescribiendo la tesis)
34k+2 34 +2 43k+1 43 +0 elemento neutro de (Z; +) ; 0 = 2 34 43k+1 2 34 43k+1
34k+2 34 + 2 34 43k+1 + 2 43k+1 43 2 34 43k+1
4 4k+2 3k+1 3k+1 3 4
3 3 +2 4 +2 4 4 3 (Prop. asociativa y distributiva)
34 34k+2 +2 43k+1 +2 43k+1 ( 17) (17 divide a ambos sumandos, luego 17 divide a la suma.)
) 17j 34(k+1)+2 + 2 43(k+1)+1

3. Probar que, si 2n 1 es primo, entonces n es primo.

Demostracin.
Procederemos por contrarrecproco, esto es:
Si n es compuesto, entonces 2n 1 es compuesto.
Como n es compuesto entonces n = n0 p; adems 2n 1 = 2n 1n ; luego
podemos factorizar como sigue:
n0 n0 n0 n0
2n 1n = 2(n0 p) 1(n0 p) = (2p ) (1p ) = (2p ) (1p ) y apartir de
esto ltimo tenemos
h i
n0 n0 n 1 n0 1 n0 2 2 n0 1
(2p ) (1p ) = (2p 1p ) (2p ) 0 + (2p ) (1p ) + (2p ) (1p ) + + (2p ) + (1p )
luego el factor (2p 1p ) > 1; de aqu se deduce que 2n 1 es compuesto
porque admite dicha descomposicin.

4. Sean a; b; c tres nmeros enteros positivos tales que 2a + 2b = 2c : Demostrar


que a = b:

Demostracin. Tmese 2c y apliquemos la denicin de potencia as:

2c = (2 2 2 2) = 2 (2 2 2 2)
| {z } | {z }
c veces k veces
Como el factor (2 2 2 2) es par, entonces puede expresarse as:
| {z }
k veces
0 1
k veces k veces
z }| { z }| { k veces k veces
B (2 2 2 2) C
B 2) (2 2 2 C z }| { z }| {
2c = (2 2 2 2) = 2 B + C = (2 2 2 2)+(2 2 2 2)
| {z } @ 2 2 A
c veces

luego, queda demostrado que la nica forma en que se puede escribir una
potencia de dos como suma de potencias de dos, es que el exponente de los
sumandos sea el mismo, de aqu que a = b: En general, este resultado puede
escribirse como: 2c 1 + 2c 1 = 2c

Por ejemplo. 24 + 24 = 16 + 16 = 32 = 25

38
21n+4
5. Probar que 14n+3 es irreducible para todo n:

Solucin. Si 21n+4
14n+3 es irreducible, esto signica que mcd 21n + 4 y 14n + 3
es1:
Procediendo segn teorema 2.2.2 tenemos

21n + 4 = (14n + 3) (1) + (7n + 1)


14n + 3 = (7n + 1) (2) + 1

As el ltimo resto distinto de cero es 1; por tanto (21n + 4; 14n + 3) = 1 y


21n+4
de aqu se sigue que 14n+3 sea irreducible.

6. Determinar el mximo comn divisor de 210 y 495, y expresarlo como una


combinacin lineal de ambos.

Solucin. Usando el algoritmo de Euclides.

495 = 2 210 + 75
210 = 2 75 + 60
75 = 1 60 + 15
60 = 15 4 + 0

Entonces el ltimo resto distinto de cero es el mcd; (210; 495) = 15:

Ahora:
15 = 75 (1 60)
60 = 210 (2 75)
75 = 495 (2 210)

A partir de esto podemos escribir las siguientes igualdades:

15 = [495 (2 210)] [210 (2 75)]


= 495 + ( 2 210 210) + 2 75
= 495 210 (2 + 1) + 2 (495 (2 210))
= 495 3 210 + 2 495 4 210
= (495 + 2 495) + ( 3 210 4 210)
= 3 (495) 7 (210)

7. Existen enteros a; b tal que sumados den 500 y (a; b) = 7 ?

Solucin. No. Si (a; b) = 7; entonces 7ja y 7jb; de aqu que 7ja + b: Pero
7 - 500

39
8. Los nmeros n y n + 1 tiene algn divisor comn distinto de 1?

Solucin. No. Algn divisor comn de n y n+1 debe dividir a (n + 1) n =


1: As dos nmeros consecutivos siempre son coprimos.

9. Demuestre que 4n + n4 no es primo si n > 1

Demostracin. Si n es par, es claro que 4n + n4 es par y mayor que 2,


luego no es primo.
Para el caso n impar utilizamos la identidad de Sophie Germain.
p p
x4 + y 4 = x2 + y 2 + x2 + y 2
2 2
2xy 2xy
p 4
Entonces, si n = 2k + 1; 4n = 42k+1 = 42k 4 = 2 2 2k ; as que podemos
escribir p 4
4n + n4 = 2 2 2k + n4 = 2n + n2 + 2k+1 n 2n + n2 2k+1 n
Comprobemos nalmente que el menor de los dos factores anteriores no es
igual a 1.
2
2n + n2 2k+1 n = 22k+1 + (2k + 1) 2k+1 (2k + 1) = 2 22k 2
2 2
22k (2k + 1) + (2k + 1) = 2k (2k + 1) + 22k 5 pues k > 0:

10. Hallar todos los nmeros naturales n tales que

n2 + 1; n2 + 3; n2 + 7; n2 + 9; n2 + 15

sean todos primos.


Solucin.
N 1 = n2 + 1 N 2 = n2 + 3 N 3 = n2 + 7 N 4 = n2 + 9 N 5 = n2 + 15
Veamos que el nico natural vlido es n = 2
Claramente n = 1 no es solucin, pues slo sera primo N1 :

Si n = 2; N1 = 5; N2 = 7; N3 = 11; N4 = 13; N5 = 19 que son todos primos.


Evidentemente, si los cinco nmeros primos han de ser primos n ha de ser
par, pues todos los primos mayores que dos son impares.

Examinemos los nmeros pares n > 2 elevados al cuadrado:

Si n acaba en 2, n2 acaba en 4 con lo que N1 acaba en 5; siendo mltiplo


de 5. N1 no es primo.
Si n acaba en 4, n2 acaba en 6 con lo que N4 acaba en 5, siendo mltiplo
de 5. N4 no es primo
Si n acaba en 6, n2 acaba en 6 con lo N4 acaba en 5, siendo mltiplo de
5. N4 no es primo.

40
Si n acaba en 8, n2 acaba en 4 con lo que N1 acaba en 5, siendo mltiplo
de 5. N1 no es primo
Si n acaba en 0, n2 acaba en 0 con lo que N5 acaba en 5, siendo mltiplo
de 5. N5 no es primo.

Por tanto, sea cual sea el valor par de n > 2 siempre encontraremos al menos
uno de los 5 nmeros mltiplo de 5.

11. (divisibilidad por 2) El entero N es divisible por 2 si el dgito de las unidades


es par.

Dicho en otras palabras para un entero de cuatro cifras N = abcd es


divisible por 2 si d es divisible por 2;es decir

2jd =) 2jN
Solucin.
Aunque la prueba la haremos para nmeros de 4 cifras, tiene validez general.
Escribamos el nmeroN = abcd en la forma:

N = 1000a + 100b + 10c + d

Es claro que 2 es divisor de 10; 100 y 1000, por teorema 2.1.1.c se tiene que
2j (1000a + 100b + 10c) y por hiptesis 2jd, entonces por el mismo resultado se
sigue que, 2j (1000a + 100b + 10c + d) ; es decir 2jN

12. Encontrar todos los divisores de 756:

Solucin. Del teorema 2.5.1 tenemos que 756 = 22 33 7; de aqu tenemos


que los divisores sean:

20 30 70 = 1 20 32 7 = 63 21 31 70 = 6 21 33 7 = 378 22 32 70 = 36
20 30 71 = 7 20 33 70 = 27 21 31 71 = 42 22 30 70 = 4 22 32 7 = 252
20 31 70 = 3 20 33 7 = 189 21 32 70 = 18 22 30 7 = 28 22 33 70 = 108
20 31 7 = 21 21 30 70 = 2 21 32 7 = 126 22 31 70 = 12 22 33 7 = 756
20 32 70 = 9 21 30 7 = 14 21 33 70 = 54 22 31 7 = 84

41
2.7 Dnde estn los enteros y los primos?

Los nmeros primos y la naturaleza.

No solo los seres humanos utilizamos los nmeros primos para protegernos,
existe una especie de cigarra que se vio obligada a protegerse de un cierto
parsito, y para ello no encontr mejor herramienta que los nmeros primos.

El siguiente ejemplo nos lo presenta Simon Singh en su libro El enigma de


Fermat
Las cigarras peridicas, muy especialmente la Magicicada septendecim, tiene
el ciclo vital ms largo de todos los insectos. Su nico ciclo vital empieza bajo
tierra, donde las ninfas absorben pacientemente los zumos de las races de los
rboles. Entonces, despus de 17 aos de esperar, las cigarras adultas emergen
de la tierra en gran nmero e invaden temporalmente nuestro paisaje. Unas
semanas despus se aparean, ponen los huevos y se mueren. La cuestin que
inquietaba a los zologos era: por qu el ciclo vital de las cigarras es tan largo?
Qu quiere decir que el ciclo vital sea un nmero primo de aos? Otra especie,
la Magicicada tredecim, aparece cada 13 aos, lo que indica que los ciclos vitales
que son nmeros primos dan algn tipo de ventaja para la conservacin de la
vida.

Segn una teora, la cigarra tiene un parsito que tambin recorre un ciclo
vital, y que la cigarra est intentando evitar. Si el parsito tiene un ciclo vital,
pongamos de 2 aos, entonces la cigarra quiere evitar un ciclo de vital que sea
divisible por 2, si no el parsito y la cigarra coincidirn regularmente. De esta
manera parecida, si el parsito tiene un ciclo vital de tres aos, entonces la
cigarra querr evitar un ciclo vital que sea divisible por 3, si no el parsito y la
cigarra volvern a coincidir. Al n, si quiere evitar encontrarse con su parsito,
la mejor estrategia de la cigarra es darse un ciclo de vida largo, que dure un
nmero primo de aos. Como nada dividir al 17, la Magicicada septendecim

42
raramente se encontrar con su parsito. Si el parsito tiene un ciclo de 2 aos
solo se encontrara cada 34, y si tiene un ciclo vital ms largo, de 16 aos por
ejemplo, solo se encontrarn cada 272 aos.

El parsito, en su lucha por sobrevivir, solo tiene dos ciclos vitales que in-
crementan las frecuencias de las coincidencias: El ciclo anual o el mismo de 17
aos que la cigarra. Ahora bien, es poco probable que el parsito pueda sobre-
vivir y reaparecer 17 aos seguidos, porque durante las primeras 16 apariciones
no habr cigarras a las cuales parasitar. De otro modo, si quieren conseguir el
ciclo de 17 aos, las generaciones de parsitos tendrn que evolucionar primero
durante un ciclo vital de 16 aos. Esto signicara que, en algn estadio evolu-
tivo de su vida, el parsito y la cigarra no coincidiran durante 272 aos! En
cualquier caso el largo ciclo vital de las cigarras y el nmero primo de aos, las
protege.

Esto podra explicar por qu el supuesto parsito no ha sido encontrado


nunca! En la lucha por coincidir con la cigarra, el parsito probablemente ha
continuado alargando su ciclo vital hasta conseguir traspasar la barrera de los
16 aos. Entonces dejar de coincidir durante 272 aos; mientras tanto, su falta
de coincidencia con la cigarra lo habr llevado a la extincin. El resultado es
una cigarra con el ciclo vital de 17 aos; ciclo que ya no le hace falta porque su
parsito ya no existe.

Algo sobre la distribucin de los primos.

Cuenta la leyenda que el matemtico y fsico polaco Stanislaw Marcin Ulam


(1909-1981) (quien trabaj en el proyecto Manhattan), asista a una aburrida
conferencia en 1963, entonces garabateando en una hoja de papel, dispuso la
serie de nmeros naturales en forma de una espiral; empezando con el 1, a la
derecha el 2, hacia arriba el 3, a la izquierda de este el 4, ms a la izquierda el
5, luego hacia abajo el 6, etc. Como se muestra en la gura.

43
Posteriormente destac en este arreglo a los nmeros primos y observ cierto
patrn, que cumplan los primos en la espiral, estos tienden a aparecer en diag-
onales alternas y en las que no hay primos estn los nmeros pares.

44
2.8 Ejercicios Propuestos

1. Determnese cules de las siguientes armaciones son verdaderas y cules


falsa.
a. Si un nmero es divisible entre 6, es divisible entre 3___
b. Si un nmero es divisible entre 3, es divisible entre 6___

c. Si un nmero es divisible entre 2 y divisible entre 3, es divisible entre 6___


d. Si un entero no es divisible entre entre 6, no es divisible entre 9___
e. Si un nmero es divisible entre 6, no es divisible entre 9___
2. Demustrese que la suma de los cubos de tres nmeros naturales sucesivos
es divisible por 9.
3. Prubese que la suma 11n+2 + 122n+1 es divisible por 133 cualquiera que sea
el nmero entero n 0
4. Prubese las siguientes armaciones
i. 6j2n3 + 3n2 + n
ii. 169j33n+3 26n 27
iii. 72n+1 48n 7 es divisible por 288
iv. 3 52n+1 23n+1 es divisible por 17
5. Demustrese la regla de divisibilidad por 3. El entero N es divisible por 3 si
la suma de sus dgitos es divisible por 3. Dicho de otra manera para un
entero de cuatro cifras
N = abcd es divisible por 3; si a + b + c + d es divisible por 3, es decir

3j(a + b + c + d) ) 3jN

6. Establzcase una demostracin o un contraejemplo para cada una de las


armaciones siguientes.

i. Si an jbn =) ajb
ii. Si nn jmm =) njm
iii. Si an j2bn y n > 1 =) ajb
7. Prubese los incisos d y e del teorema 2.2.4.

8. Prubese la proposicin 2.3.1

45
9. Prubese el teorema 2.2.7
10. Encuntrese el mcd de 576 y 73 y nmeros x y y tales que 576x + 73y =
(576; 73)
11. Hllese dos nmeros sabiendo que su mximo comn divisor es 120 y la
diferencia de sus cuadrados 345600
12. Hllese dos nmeros naturales sabiendo que su producto es 3024 y su mn-
imo comn mltiplo 504.
13. Determnese dos nmeros naturales cuyo mximo comn divisor es 18, sa-
biendo que uno de ellos tiene 21 divisores y el otro tiene 10.
14. Si dos nmeros son coprimos y su mnimo comn mltiplo es 22829, hllese
dichos nmeros.
15. Prubese que no existen los nmeros x y y que satisfagan x + y = 100 y
(x; y) = 3
16. Encuntrese los valores de x y y (si existen) que satisfagan.
i. 71x 50y = 1
ii. 43x + 64y = 1
iii. 93x 81y = 3
17. Dados dos nmeros primos distintos p y q, encuntrese el nmero de difer-
entes divisores positivos de:

a. pq b. p2 q c. p2 q 2 d. pn q m

18. Encuntrese el menor nmero natural n tal que n! es divisible por 990

19. Prubese que si un nmero tiene un nmero impar de divisores, entonces


este es un cuadrado perfecto.
p
20. Probar que si n es compuesto tiene un divisor primo que satisface p n

46
3 Nmeros de Fibonacci

Es como preguntar por qu la novena sinfona de Beethoven es bella. Si no ve


que es as, nadie se lo puede explicar. Yo s que los nmeros son bellos. Si no
lo son nada lo es.
Paul Erds.

El conjunto de los nmeros de Fibonacci es muy particular por encontrarse


vinculado a muchas de las actividades del hombre, sobre todo aquellas que
implican armona. En este captulo estudiaremos las principales caratersticas
aritmticas de estos nmeros como: la divisibilidad, mximo comn divisor,
paridad entre otras. Tambin mostraremos algunas de sus aplicaciones.

3.1 La Sucesin de Fibonacci

Existe un conjunto de nmeros muy particular y con mucha belleza, estos


nmeros son conocidos como nmeros de Fibonacci en honor a su descubri-
dor Leonardo de Pisa, alias Fibonacci (hijo de Bonacci). Leonardo hizo muchos
aportes notables a las matemticas especialmente en la aritmtica, alrededor del
ao 1202 escribi el libro sobre el baco. Este era una inmensa obra compiladora
de los conocimientos matemticos de los pueblos que vivan en las costas del
Mediterrneo. En este libro se encuentra un problema que revel el canon de la
naturaleza:

Cuntas parejas de conejos nacen, en el transcurso de un ao, de una


pareja inicial?

Probablemete alguien observ la naturaleza reproductoria de los conejos y


obtuvo las siguientes premisas: Cada pareja produce otra al cabo de un mes y
una pareja inicial de conejos puede parir a los dos meses de haber nacido. De
esto y con el supuesto de un rea cercada, podemos deducir que:

A partir de una pareja de conejos bebs en el primer y segundo mes ten-


dramos un par de conejos, pues la hembra ser adulta hasta el segundo mes,
donde podr reproducirse; tendra un par de bebs en el tercer mes, as sern
dos pares de conejos, luego en el cuarto mes los bebs habrn crecido y repro-
ducido y tendremos tres parejas de conejos; los originales y sus dos cras y las
primeras cras de estos. Siguiendo este razonamiento encontramos que para el
duodcimo mes tendremos 377 parejas de conejos.

As encontramos un conjunto de nmeros enteros muy particulares estos son:


1; 1; 2; 3; 5; 8; 13; 21; 34 : : : esta se conoce como sucesin de Fibonacci donde cada
trmino es la suma de los dos anteriores, as esta es una sucesin recurrente.

47
Pasemos ahora de los conejos a los nmeros y consideremos la sucesin
numrica

3.1.1 Denicin. La sucesin (vn ) ; llamada de Fibonacci, cuyos trminos son


1; 1; 2; 3; 5; 8; 13; 21; 34 : : : y en la cual cada trmino es la suma de los dos
inmediatos anteriores, est denida por

vn = vn 1 + vn 2 ; con v1 = v2 = 1
Los nmeros de Fibonacci poseen una serie de propiedades interesantes e
importantes, las cuales veremos en este captulo.

Enpecemos calculando la suma de los n primeros nmeros de Fibonacci. As


podemos enunciar el teorema siguiente

3.1.1 Teorema. La suma de los primeros n nmeros de Fibonacci esta dada


por
Xn
vi = v1 + v2 + + vn = vn+2 1
i=1

Demostracin. Claramente, tenemos de la denicin de la sucesin de


Fibonacci que

v1 = v3 v2
v2 = v4 v3
.. .. ..
. . .
vn 1 = vn+1 vn
vn = vn+2 vn+1

Sumando miembro a miembro estas igualdades, encontramos

v1 + v2 + + vn = v3 v2 + v4 v3 + vn+1 vn + vn+2 vn+1

v1 + v2 + + vn = vn+2 1 (recordando que v2 = 1)


n
X
Por tanto, vi = v1 + v2 + + vn = vn+2 1
i=1

Hemos denido los nmeros de Fibonacci mediante la ecuacin recurrente,


es decir, empleando la induccin segn el ndice. Pero resulta que todo nmero
de Fibonacci puede denirse de un modo ms directo, esto es, como funcin de
su ndice.

48
Con este n, observemos el comportamiento de las distintas sucesiones v1 ; v2 ; : : : ; vn ; : : :
que satisfacen la ecuacin

vn = vn 1 + vn 2 (1)
Diremos que todas estas sucesiones son soluciones de la ecuacin (1) : De
aqu en adelante indicaremos por V; V 0 y V 00 las sucesiones

v1 ; v2 ; v3; : : :
v10 ; v20 ; v3;
0
:::
00 00 00
v1 ; v2 ; v3; : : :

3.1.1 Lema. Si V es una solucin de la ecuacin (1) y c es una constante, tam-


bin la sucesin cV (es decir, la sucesin cv1 ; cv2 ; cv3; : : :) es una solucin
de esta ecuacin.

Demostracin. Multiplicando por c ambos miembros de la igualdad

vn = vn 1 + vn 2

obtenemos
cvn = cvn 1 + cvn 2

lo que prueba el lema.

3.1.2 Lema. Si las sucesiones V 0 y V 00 son soluciones de la ecuacin (1) ;


Tambin la suma V 0 +V 00 (esto es, v10 + v100 ; v20 + v200 ; v30 + v300 : : :) es solucin
de esta ecuacin.

Demostracin. Por hiptesis, tenemos que

vn0 = vn0 1 + vn0 2 y vn00 = vn00 1 + vn00 2

Sumando estas igualdades miembro a miembro, encontramos

vn0 + vn00 = vn0 1 + vn00 1 + vn0 2 + vn00 2

la ltima igualdad prueba el lema.

Consideremos ahora V 0 y V 00 dos soluciones no proporcionales de la ecuacin


(1) ( es decir, dos soluciones de la ecuacin (1) tales que cualquiera que sea la
v0
constante c habr un nmero n para el que vn00 6= c).
n

49
3.1.1 Proposicion. Toda sucesin V; solucin de la ecuacin (1) ; puede ser
representada as

V = c1 V 0 + c2 V 00 (2)

donde c1 y c2 son constantes. A la ecuacin (2) se estila llamar solucin


general de la ecuacin (1) :

Probaremos primero que siendo V 0 y V 00 dos soluciones no proporcionales


de la ecuacin (1) ; se tiene

v10 v0
00 6= 002 (3)
v1 v2
(La no proporcionalidad es visible ya en los primeros trminos de las sucesiones V 0 y V 00 )

Demostracin (3). Supongamos que para dos soluciones no proporcionales


V 0 y V 00 de la ecuacin (1) se tiene

v10 v20
=
v100 v200
Formemos la siguiente proporcin
v10 + v20 v20
=
v100 + v200 v200

recordando que V 0 y V 00
son soluciones de la ecuacin (1) ;

v30 v20
=
v300 v200
Anlogamente comprobamos (haciendo induccin en n !) que

v30 v0 vn0
00 = 004 = = =
v3 v4 vn00

Si esto ocurre, se tiene que V 0 y V 00


son proporcionales lo que es absurdo,
luego (3) es verdadera.

Tomemos ahora una sucesin V; solucin de la ecuacin (1) : Segn hemos


visto ya al inicio de este apartado, esta sucesin queda perfectamente determi-
nada si se indican sus dos primeros trminos v1 y v2 :
Busquemos los valores de c1 y c2 de modo que sea

c1 v10 + c2 v200 = v1
(4)
c1 v10 + c2 v200 = v2

50
La suma c1 V 0 + c2 V 00 coincidir con V , esto lo garantiza los lemas 3:1:1 y
3:1:2: El sistema de ecuaciones (4) tiene solucin respecto a c1 y c2 en virtud de
la proposicin 3.1.1, para cualesquiera que sean los nmeros v1 y v2 :

v1 v200 v2 v100 v10 v2 v20 v1


c1 = y c2 =
v10 v200 v100 v20 v10 v200 v100 v20

Sustituyendo en (2) los valores obtenidos para c1 y c2 encontramos la repre-


sentacin requerida para la sucesin V:

Es decir, para describir todas las soluciones de la ecuacin (1) basta encon-
trar dos soluciones no proporcionales de la misma. Encontremos estas soluciones
entre las progresiones geomtricas. De acuerdo al lema 3:1:1, basta considerar
las progresiones cuyos primeros trminos son 1. Tomemos la progresin

1; x; x2 ; : : :
Para que sea una solucin de la ecuacin (1) ; es suciente que para todo n
se cumpla la igualdad

xn 2
+ xn 1
= xn
dividiendo por xn 2
;
1 + x = x2 (5)
p p
1+ 5 1 5
Las races de esta ecuacin, es decir, los nmeros 2 y 2 ; sern las
razones buscadas de las progresiones.

Llammoslas por y ; respectivamente. Los nmeros y ; como races


de la ecuacin (5) ; satisfacen las relaciones 1 + = 2 , 1 + = 2 y = 1:
As hemos encontrado dos progresiones geomtricas, soluciones ambas de
(1) : Por eso, toda sucesin del tipo
2 2
c1 + c2 ; c1 + c2 ; c1 + c2 ;::: (6)
Son soluciones de la ecuacin (1) : Adems las progresiones encontradas
tienen distintas razones y, por ende, no son proporcionales, esto es, la frmula
(6) debe coincidir con la sucesin de Fibonacci.
Para ello, como hemos explicado, hay que determinar c1 y c2 de las ecua-
ciones

c1 + c2 = v1 y c1 + c2 = v2 ;
Es decir, del sistema

c1 + c2 = 1
p p
1+ 2 5 1 2
5
c1 2 + c2 2 = 1

51
Resolvindolo, encontramos
p p !
1+ 25 1 2
5
c1 = p y c2 = p
225 225
de manera que
n 1 n 1
vn = c1 + c2
p p n 1 p p n 1
1+p2 5 1+ 5 2
1 p2 5 1 2
5
= 225 2 225 2

es decir,
p n p n
1+ 2 5 1 2
5
2 2
vn = p
2
5
Esta ltima expresin lleva el nombre de frmula de Binet en memoria del
matemtico que la encontr (Jacques Philippe Marie Binet a mediados del siglo
XIX).

3.1.2 Teorema. El nmero de Fibonacci vn es el entero ms prximo al


n
nmero p 2 , o sea, es el entero ms prximo al n-simo trmino an de la
5
progresin geomtrica cuyo primer trmino es p 2
5
y cuya razn es :

Demostracin. Basta demostrar que el valor absoluto de la diferencia entre


vn y an es siempre menor que 21 : Esto es
n n n n n n n
j j
jvn an j = p
2
p
2
= p
2
= p2
5 5 5 5
n
Puesto que = 0:618033 : : : ; se tiene j j < 1; es decir j j < 1 para todo
jn p
n; con mayor razn jp2
5
< 1
2 ya que 2
5 > 2: lo que prueba el teorema.
Por ejemplo, calculemos para n = 13
p 13
1+ 2 5
13 2 521:0019
p
2
= p
2
= = 232:9957
5 5 2: 236 1

El nmero entero ms prximo a 232:9957 es 233 que corresponde a v13 en


la sucesin de Fibonacci.

52
3.2 Propiedades Aritmticas de los Nmeros de Fibonacci.

Estudiaremos algunas propiedades sobre la divisibilidad, mximo comn divisor


y otras caracterizaciones aritmticas de los nmeros de Fibonacci.

3.2.1 Lema. Probar la vlidez de la siguiente frmula para los nmero de


Fibonacci
vn+m = vn 1 vm + vn vm+1

3.2.1 Teorema. Si n es divisible por m; tambin vn es divisible por vm :

Demostracin. Supongamos que n es divisible por m; esto es, n = mk:


Haremos nuestra demostracin haciendo induccin en k:
Para k = 1 se tiene n = m y es evidente que vn es divisible por vm : Supong-
amos que vn=mk es divisible por vm y consideremos vm(k+1) : Pero vm(k+1) =
vmk+m ; en virtud del lema 3.2.1
vm(k+1) = vmk+m = v(mk) 1 vm + vmk vm+1
Es claro que vm divide el primer sumando del tercer miembro. El segundo
sumando es mltiplo de vmk ; esto es, tambin es divisible por vm segn la
hiptesis inductiva. De aqu se deduce que la suma de estos dos sumando, o
sea, vm(k+1) ; es divisible por vm : Que era lo que queriamos demostrar.

Por ejemplo, tomemos m = 5 y n = 15; es claro que mjn: Por otra parte
los nmeros vm = v5 = 5 y vn = v15 = 610; tambin es claro que vm jvn : Puesto
que v15 = v5 122:

3.2.2 Teorema. Los nmeros de Fibonacci consecutivos son coprimos.

Demostracin. Supongamos, en contra de la armacin que vn y vn+1


tienen un divisor comn d > 1: La diferencia vn+1 vn es divisible por d: Pero
como vn+1 vn = vn 1 ; resulta que d divide tambin vn 1 : Anlogamente se
demuestra (haciendo induccin!) que d divide vn 2 ; vn 3 ; : : : ; etc: y nalmente
a v1 : Pero v1 = 1 y no puede ser divisible por d > 1: Por tanto los nmero vn y
vn+1 son coprimos.

Ejemplos
mcd (144; 233) = 1
mcd (21; 34) = 1

Observemos los siguientes casos

53
(1; 2) = 1; (2; 8) = 2; (3; 21) = 3; (5; 55) = 5

Cabe hacernos la siguiente pregunta, El mximo comn divisor de dos


nmeros no consecutivos de Fibonacci, es otro nmero de Fibonacci?

3.2.3 Teorema. Para los nmeros de Fibonacci, tiene lugar la igualdad sigu-
iente
(vm ; vn ) = v(m;n)

Esto es, el mximo comn divisor de dos nmeros de Fibonacci es el nmero


de Fibonacci que corresponde al mcd de los ndices de los nmeros dados.

Demostracin. Supongamos que m > n y apliquemos el algoritmo de


Euclides a los nmeros m y n:

m = nq0 + r1 donde 0 r1 < n


n = r 1 q 1 + r2 donde 0 r2 < r 1
r 1 = r 2 q 2 + r3 donde 0 r3 < r 2
.. .. ..
. . .
rt 2 = rt 1 q t 1 + rt donde 0 rt < r t 1
rt 1 = r t q t

Luego por teorema 2.2.2 resulta que rt es el mximo comn divisor de m y


n: Puesto que m = nq0 + r1 ; resulta que

(vm ; vn ) = (vnq0 +r1 ; vn )


esto es, por el lema 3.2.1,

(vm ; vn ) = (vnq0 r1 vr1 + vnq1 vr1 +1 ; vn )

recordando que (a; b) = (a + c; b) ; podemos escribir

(vm ; vn ) = (vnq0 r1 vr1 ; vn )

de igual forma teniendo presente que (a; bc) = (a; b) ; se sigue

(vm ; vn ) = (vr1 ; vn )

Anlogamente podemos escribir que

(vr1 ; vn ) = (vr2 ; vr1 )


(vr2 ; vr1 ) = (vr3 ; vr2 )
.. .. ..
. . .
vrt 1 ; vrt 2
= vrt ; vrt 1

54
Comparando estas igualdades, encontramos

(vm ; vn ) = vrt ; vrt 1

Como rt divide a rt 1 ; luego por el teorema 3.2.1, debe ser que vrt jvrt 1 ; debe
ser por lo tanto vrt ; vrt 1 = vrt y recordando, nalmente, que rt = (m; n) ;
obtenemos (vm ; vn ) = v(m;n)

3.2.1 Proposicin. Un nmero de Fibonacci es par si, y slo si, su ndice es


divisible por 3, esto es
2jvn () 3jn

Demostracin. ((=) Si 3jn =) 2jvn : Si 3jn entonces n = 3k; luego


haciendo induccin en k; resulta para k = 1; n = 3 y v3 = 2 es claro que 2jv3 :
Asumamos la validez de 2jv3k para algn k y probemos la validez para k + 1;
esto es, 2jv3(k+1) : Si tenemos n = 3 (k + 1) ; entonces vn = v3(k+1) = v3k 1 v3 +
v3k v4
recordando que v3 = 2; tenemos vn = v3(k+1) = 2v3k 1 + v3k v4 ; es claro que
el primer sumando es par, y el segundo es divisible por 2 por hiptesis inductiva.
Luego 2jv3(k+1) :

(=)) Si 2jvn =) 3jn: Haremos esta prueba por contrarrecproco, esto es,
si 3 - n =) 2 - vn : Si 3 - n, entonces n = 3k + r donde 0 < r < 3: Haciendo
induccin sobre k; tenemos para k = 1
n = 3 + r; pero r solo puede ser 1 o 2, lo cual nos da los casos v4 = 3 y
v5 = 5 que en ningn caso es par. Suponemos la validez de 3 - n = 3k+r =) 2 -
vn=3k+r para algn k: y probemos la validez para k+1: Luego si n = (3k + 1)+r;
entonces podemos escribir

vn = v(3k+1)+r = v(3k+r)+3 = v(3k+r) 1 v3 + v3k+r v4


recordemos que v3 = 2 y v4 = 3; as tenemos

vn = 2 v(3k+r) 1 + 3 v3k+r

como 2 - 3 y 2 - v3k+r esto garantiza que 2 - vn :

Por ejemplo, si n = 9 tenemos v9 = 34 y 2j34:


Por otro lado, 2jv12 = 144 y 3j12:

55
3.3 Nmeros de Fibonacci y Las Fracciones Continuas

Consideremos la expresin
1
q0 + 1 (1)
q1 + q2 + 1
..
. + q1n

donde q1 ; q2 ; : : : ; qn son enteros positivos y q0 es un entero no negativo, esto


es q0 puede se cero.

Las expresiones del tipo (1) se denominan fracciones continuas y el proceso


de conversin de un nmero en una fraccin continua se denomina desarrollo en
fraccin continua.

Aprendamos cmo obtener los cocientes incompletos de este desarrollo para


el caso de una fraccin ordinaria ab :
Consideremos para este n el algoritmo de Euclides aplicado a los nmeros
a y b:

a = bq0 + r1 donde 0 r1 < b


b = r1 q 1 + r2 donde 0 r2 < r1
r1 = r 2 q 2 + r 3 donde 0 r3 < r2
.. .. .. (2)
. . .
rn 2 = rn 1 q n 1 + rn donde 0 rn < rn 1
rn 1 = rn q n

De la primera igualdad es claro que


a r1 1
= q0 + = q0 + b
b b r 1

Pero de la segunda igualdad del sistema (2) se deduce que

b r2 1
= q1 + = q1 + r1
r1 r1 r2

y Ahora teniendo presente la tercera igualdad del sistema (2)


r1 r3 1
= q2 + = q2 + r2
r2 r2 r3

Tomando en cuenta estas igualdades y haciendo las sustituciones adecuadas


obtenemos

56
a 1
= q0 +
b q1 + q +1 r1
2 2
r3

continuando este proceso hasta el n resulta obvia la expresin


1
q0 + 1
q1 + q2 + 1
..
. + q1n

3.3.1 Teorema. Los concientes incompletos correspondientes de dos fracciones


continuas iguales, son iguales

Demostracin. Tomemos dos fracciones continuas y 0 : Sean q0 ; q1 ; q2 ; : : :


y q00 ; q10 ; q20 ; : : : sus cocientes incompletos respectivamente. Probemos que la
igualdad = 0 implica las igualdades q0 = q00 ; q1 = q10 ; q2 = q20 ; etc. En
efecto, q0 es la parte entera del nmero y q00 es la parte entera de 0 ; de aqu la
nica posibilidad es que q0 = q00 : Ahora bien, podemos representar las fracciones
continuas y 0 en la forma
1 1
q0 + y q00 + 0
1 1

donde 1 y 01 tambin son fracciones continuas. Puesto que = 0 y q0 = q00 ;


debe ser 1 = 01 : Pero en tal caso son iguales las partes enteras de los nmeros
0 0
1 y 1 ; o sea, q1 y q1 : Continuando este razonamiento encontramos que q2 =
0 0
q2 ; q 3 = q3 ; : : :

Sea
1
= q0 + 1
q1 + q2 + 1
..
. + q1n
una fraccin continua. Consideremos los nmeros
1 1
q0 ; q 0 + ; q0 + 1 ;:::
q1 q1 + q2

estos nmeros expresados como fracciones irreducibles


P0 q0
Q0 = 1
P1
Q1 = q0 + q11
P2
Q2 = q0 + q +1 1
1 q2
.. .. ..
. . .
Pn
Qn =

57
se denominan reducidas de la fraccin continua : De la secuencia anterior
Pk+1 Pk
se ve que Q k+1
se obtine de Q k
sustituyendo el nico cociente incompleto de
esta reducida, o sea, qk ; por qk+1 :

3.3.1 Lema. Para toda fraccin continua se cumplen las relaciones siguientes

Pk+1 = Pk qk+1 + Pk 1 (1)


Qk+1 = Qk qk+1 + Qk 1 (2)
k
Pk+1 Qk Pk Qk+1 = ( 1) (3)

Haremos la demostracin del lema 3.3.1 probando simultneamente las tres


igualdades y aplicando induccin sobre k.
Para k = 1: Tenemos:
P1 1 q0 q1 + 1
= q0 + =
Q1 q1 q1

Puesto que los nmeros q0 q1 + 1 y q1 son coprimos, la fraccin q0 qq11+1 es


P1
irreducible; al mismo, la fraccin Q1
es irreducible. Pero los numeradores y
los denominadores de dos fracciones irreducibles iguales son iguales. Esto es
P1 = (q0 q1 + 1) y Q1 = q1 :
Tenemos, luego,

P2 1 q0 (q1 q2 + 1) + q2
= q0 + 1 =
Q2 q1 + q2
q1 q2 + 1

recordando aqu que; (a; bc) = (a; b) = (a + c; b) tenemos

(q0 (q1 q2 + 1) + q2 ; q1 q2 + 1) = ((q1 q2 + 1) + q2 ; q1 q2 + 1) = (q2 ; q1 q2 + 1)

y por la misma razn pasa que

(q2 ; q1 q2 + 1) = (q2 ; 1) = 1
P1 q0 q1 +1
de aqui se siguie que Q1 = q1 sean irreducibles, de modo que

P2 = q0 (q1 q2 + 1) + q2 = (q0 q1 + 1) q2 + q0 = P1 q2 + P0
y
Q2 = q1 q2 + 1 = Q1 q2 + Q0
nalmente la igualdad
1
P2 Q1 P1 Q2 = (q0 (q1 q2 + 1) + q2 ) (q1 ) (q0 q1 + 1) (q1 q2 + 1) = ( 1)

58
Hasta aqu tenemos la validez para k = 1; y la base de la induccin para
algn entero k; bien ahora consideremos el caso para k + 1:
Consideremos la fraccin
Pk+1 Pk qk+1 + Pk 1
=
Qk+1 Qk qk+1 + Qk 1

Pk+2 Pk+1
Como hemos dicho ya Q k+2
se obtine de Q k+1
sustituyendo en sta qk+1
1
por qK+1 + qk+2 ; puesto que qk+1 no gura en las frmulas para Pk ; Qk ; Pk 1 y
Qk 1 ; tenemos

1
Pk+2 Pk qK+1 + qk+2 + Pk 1
=
Qk+2 Qk qK+1 + 1
+ Qk
qk+2 1

recordando las hiptesis inductivas (1) y (2)


Pk+2 Pk+1 qk+2 + Pk
= (4)
Qk+2 Qk+1 qk+2 + Qk
Demostraremos que el segundo miembro de (4) es una fraccin irreducible,
para ello basta probar que su numerador y denominador son coprimos.
Supongamos que los nmeros Pk+1 qk+2 + Pk y Qk+1 qk+2 + Qk poseen un
divisor comn d > 1: En este caso, la expresin

(Pk+1 qk+2 + Pk ) Qk+1 ( Qk+1 qk+2 + Qk ) Pk+1


tambin ser divisible por d: Pero, segn la hiptesis inductiva (3) ; esta
k+1
expresin es igual a ( 1) y d no puede dividirla.
Por lo tanto, el segundo miembro de (4) es irreducible de modo que (4) es
una igualdad entre dos fracciones irreducibles. Luego,

Pk+2 = Pk+1 qk+2 + Pk y Qk+2 = Qk+1 qk+2 + Qk


Para nalizar la demostracin falta demostrar que
k+1
Pk+2 Qk+1 Pk+1 Qk+2 = ( 1)

Pero de los resultados ya obtenidos

Pk+2 Qk+1 Pk+1 Qk+2 = (Pk+1 qk+2 + Pk ) Qk+1 (Qk+1 qk+2 + Qk ) Pk+1
= Qk+1 Pk+1 qk+2 + Pk Qk+1 Pk+1 Qk+1 qk+2 Qk Pk+1
= (Qk+1 Pk+1 qk+2 ) + (Pk Qk+1 Qk Pk+1 ) (Pk+1 Qk+1 qk+2 )
= (Qk Pk+1 + Pk Qk+1 ) ( 1)
k+1
= ( 1)

con lo cual queda demostrado el lema.

59
3.3.2 Teorema. Si una fraccin incompleta tiene n cocientes incompletos,
todos iguales a 1; esta fraccin es igual a vn+1
vn

Demostracin. Sea n la fraccin continua de n cocientes incompletos


iguales a 1. Podemos escribir entonces

1; 2; 3; : : : ; n

las fracciones reducidas de la fraccin n :


Sea
Pk
k =
Qk
Puesto que
1 1
1 =1= y 2 =1+ =2
1 1
debe ser P1 = 1 y P2 = 2: Adems Pn+1 = Pn qn+1 + Pn 1 ; por lo probado
en el lema 3.3.1; como todos los cocientes son iguales a 1; se tiene que qn+1 = 1:
Por tanto podemos escribir

Pn+1 = Pn + Pn 1

de donde tenemos que

Pn = Pn 1 + Pn 2

esta ltima frmula coincide con la denicin para los nmeros de bonacci,
por tal razn
Pn = vn+1
Anlogamente tenemos Q1 = 1; Q2 = 1 y Qn+1 = Qn qn+1 + Qn 1
luego
Qn+1 = Qn + Qn 1
de modo que Qn = vn : Por consiguiente
vn+1
n =
vn

Toda la discusin sobre fracciones continuas nitas, es aplicable de forma


natural al caso de fracciones continuas innitas.

Determinemos el valor de la fraccin continua innita


1
1+ 1
1+ 1+ 1
..
.+ 1
..
. +1

60
sabemos ya que este valor es igual a lim n ; donde n = vn+1vn : Calculemos
n*1
este lmite
n
Por el teorema 3.1.2 sabemos que vn es el entero ms prximo a p2 ; es decir,
5
para todo n se tiene
n
1
vn = p
2
+ n donde j n j < :
5 2
Luego tenemos
p
n+1 p
2 n+1
2
5
vn+1 p
2 + n+1 + n+1 5 lim + n
5 n n*1
lim n = lim = lim n = lim p
2 = p
2
n*1 n*1 vn n*1 p + n*1 5 5
2
5 n 1+ n
n lim 1+ n
n
n*1
p
Pero n+1 2 5 es una magnitud acotada (su valor absoluto es menor que 2) y
n
crece indenidamente cuando n tiende al innito (porque > 1). Por tanto
p ! p
2 2
n+1 5 n+1 5
lim + n
= + lim n
=
n*1 n*1

y !
p
2
p
2
n 5 n 5
lim 1+ n
= 1 + lim n
=1
n*1 n*1

Finalmente
lim n =
n*1
p
1+ 2 5
recordemos que = 2 ; tenemos, entonces
p
1+ 25
lim n = 1: 6180
n*1 2

v5 5 v14 377
Por ejemplo, la razn v4 = 3 = 1: 666 7 y para v13 = 233 = 1: 618:

61
3.4 Dnde estn los nmeros de Fibonacci?

Fibonacci en la Naturaleza.

Fibonacci y los vegetales: El ejemplo que quizs sea el ms sencillo, es el


caso de la orientacin de las espirales en una pia, si contamos las espirales en
un sentido y luego en el otro (sentido) encontramos los nmeros 8 y 13 en otras
especies aparecen 5 y 8, en ambos casos nmeros de Fibonacci consecutivos.

Fibonacci y las ores: Si por curiosidad contamos los ptalos de una or


cualquiera que esta sea, en un 95% encontraremos nmeros de Fibonacci para
estos, as por ejemplo, las margaritas tienen en general 21 34 ptalos y algunas
tienen 55 e incluso alcanzan 89 todos nmeros Fibonacci.

Fibonacci y los animales: Adems del ya mencionado problema de los


conejos, tambin encontramos nmeros de Fibonacci en el rbol genealgico de
las abejas machos. En toda colmena existe una abeja hembra llamada reina,
que es la nica capaz de producir huevos. Las abejas obreras tambin son
hembras, pero no producen huevos, solo trabajan. En la colmena tambin exis-
ten abejas machos, que no trabajan y su nica funcin es aparear a la reina
(znganos). Estos provienen de huevos de la abeja reina no fertilizados, y por
lo tanto tienen madre, pero no tienen padre, por lo que l (1) tiene una madre
(1; 1), dos abuelos los padres de la reina (1; 1; 2), tres bisabuelos -por que
el padre de la reina no tuvo padre-(1; 1; 2; 3), cinco tatarabuelos, (1; 1; 2; 3; 5)

62
y ocho tataratatarabuelos, (1; 1; 2; 3; 5; 8), en denitiva sigue estrictamente la
sucesin de nmeros de Fibonacci.

En las dimensiones del hombre.

La relacin entre la altura de un ser humano y la altura de su ombligo.

La relacin entre la distancia del hombro a los dedos y la distancia del


codo a los dedos.
La relacin entre la altura de la cadera y la altura de la rodilla.
La relacin entre el primer hueso de los dedos (metacarpiano) y la primera
falange, o entre la primera
p
y la segunda, o entre la segunda y la tercera,
si dividimos todo es 1+2 5 .

63
3.5 Ejercicios Propuestos

1. Calclese los primeros 50 nmeros de Fibonacci.


2. Prubese el lema 3.2.1
3. Encuntrese una expresin que de la suma para los nmeros de Fibonacci de
ndices impares y prubese su validez para todo n:
3. Prubese que para los nmeros de Fibonacci es vlida la igualdad siguiente
v12 + v22 + v32 = vn vn+1

4. Prubese el siguiente teorema.

Teorema. Cualquiera que sea el nmero entero m, entre los m2 1 primeros


nmeros de Fibonacci habr al menos uno divisible por m:

5. Calclese el mcd para los siguientes conjunto de nmeros


(v7 ; v9 ) ; (v5 ; v15 ) ; (v4 ; (v7 ; v9 )) ; ((v21; v12 ) ; (v7 ; v9 ))
6. Demustrese los siguientes criterios de divisibilidad para nmeros de Fi-
bonacci
6.1 Un nmero de Fibonacci es divisible por 3 si, y slo si, su ndice es divisible
por 4.
6.2 Un nmero de Fibonacci es divisible por 4 si, y slo si, su ndice es divisible
por 6
6.3 Un nmero de Fibonacci es divisible por 5 si, y slo si, su ndice es divisible
por 5
6.4 Un nmero de Fibonacci es divisible por 7 si, y slo si, su ndice es divisible
por 8
7. Considrese los nmeros primos de Fibonacci, con ndices mayor a 3; qu
puede decirse de los ndices?
8. Desarrllese en fracciones continuas los siguientes nmeros racionales
22 355 102595 103993 1461
7 113 32657 33102 4

9. Calclese las primeros 5 trminos de la sucesin de Fibonacci usando la


frmula de Binet y prubese que sta es vlida para todo n:
10. Un saltador puede desplazarse en una sla direccin a lo largo de una franja
cuadriculada saltando cada vez a la casilla inmediata o por encima de ella
a la siguiente. cuntos modos de desplazarse en n 1 casillas y, en
particular, de la primera a la n esima tiene el saltador? (dos modos son
idnticos si en cada uno de ellos el saltador se posa en la misma casilla?

64
4 Funciones Aritmticas

Con nmeros se puede demostrar cualquier cosa.


Thomas Carlyle

En este apartado estudiaremos las funciones ms importantes de la teora


de nmero; daremos sus deniciones, algunas aplicaciones y su importancia en
la aritmtica.

4.1.1 Denicin. Una funcin real o compleja denida sobre los enteros posi-
tivos se llama una funcin aritmtica o una funcin de teora de nmeros.

Empezaremos con la funcin parte entera que ya antes habamos mencionado


en el captulo 2.

4.1.1 Teorema. Sea x y y nmeros reales. Entonces se tiene que


a. [x + m] = [x] + m; si m 2 Z
b. [x] + [y] [x + y] [x + y] + 1
0 si x es entero,
c. [x] + [ x] =
1 en cualquier otro caso
h i
[x] x
d. m = m ; si m es un entero positivo

e. x + 12 es el entero ms prximo a x: Si dos enteros son igualmente prximos


a x; es el mayor de los dos.

Demostracin. La parte a) es evidente, puesto que si m 2 Z entonces


[m] = m: De aqu que, [x + m] = [x] + m; puesto que x no es necesariamente
entero.
Para la parte b) se escribe x = n + "; y = m + ; donde m y n son enteros y
0 < 1, 0 < 1.
Entonces

[x] + [y] = n + m [n + " + m + ] = n + m + [" + ] n+m+1 [x + y] + 1

Una vez ms, para la parte c); escribiendo x = n + "; tambin se tiene
x = n 1 + 1 "; 0 < 1 " 1:
Entonces
0 si v = 0
[x] + [ x] = n + [ n 1+1 "] = n n 1 + [1 "] =
1 si v > 0

65
y se tiene c):

A modo de ejemplo, veamos una aplicacin para el teorema anterior

Probar que
n!
a1 !a2 ! ar !
es un entero. Si ai 0; a1 + a2 + a3 + + ar = n: Para hacerlo debe
demostrarse que todo nmero primo divide al numerador para la pontencia
ms alta que divide al denominador. Aplicando el teorema 2.5.2 solamente es
necesario probar
X n X a1 X a2 X ar
+ + +
pi pi pi pi

aplicando repetidamente el teorema 4.1.1b, resulta

a1 a2 ar a1 + a2 + a3 + + ar n
+ i + + =
pi p pi pi pi

Sumando esta expresin sobre i se tiene el resultado deseado.

Veamos este resultado particularmente


8 + 6 + 7 + 12 = 33; Luego
33!
; debe ser entero
8!6!7!12!
Por el teorema 2.5.2 tenemos las siguientes descomposiciones en factores
primos

33! 231 315 57 74 113 132 17 19 23 29 31


8!6!7!12! = (27 32 5 7)(24 32 5)(24 32 5 7)(210 35 52 7 11)

231 315 57 74 113 132 17 19 23 29 31


= 225 311 55 73 11

= 26 34 52 7 112 132 17 19 23 29 31

que resulta ser entero, ntese que tal como lo predijo el resultado anterior el
exponente que gura en el numerador es mayor o igual al del denominador por
cada factor primo.

66
4.1.2 Denicin. La funcin de Mbius (n) se dene como sigue:

(1) = 1;
Si n > 1; escribimos n = p1 1 pk k : Entonces
k
(n) = ( 1) si 1 = 2 = = k = 1;
(n) = 0 en otro caso.

Observemos algunos ejemplos;


1
(2) = 21 ; como 1 = 1; se tiene ( 1) = 1

1
(3) = 31 ; como 1 = 1; se tiene ( 1) = 1

(4) = 22 ; como 1 = 2 6= 1; por denicin (4) = 0


Luego

n: 1 2 3 4 5 6 7 8 9 10
(n) : 1 1 1 0 1 1 1 0 0 1

4.1.2 Teorema. Si n 1 tenemos


X 1 1 si n = 1
(d) = =
n 0 si n > 1
djn

Demostracin. La expresin es claramente cierta para n = 1; puesto que


si n = 1; d debe ser tambin 1 y por denicin de se sigue que (1) = 1:
Suponemos,
P entonces, que n > 1 y escribimos n = p1 1 pk k : En la expresin
(d) los nicos trminos no nulos proceden de d = 1 y de los divisores de n
djn
que son productos de primos distintos. Entonces
X
(d) = (1)+ (p1 )+ + (pk )+ (p1 p2 )+ + (pk 1 pk )+ + (p1 p2 pk ) ( )
djn

Ahora observemos con cuidado


k
(p1 ) + + (pk ) = ( 1)
1
k 2
(p1 p2 ) + + (pk 1 pk ) = ( 1)
2
k 3
(p1 p2 p3 ) + + (pk 2 pk 1 p k ) = ( 1)
3

67
podemos seguir con estos arreglos hasta nalizar con (p1 p2 pk ) ; susti-
tuyendo en la expresin ( ), nos resulta
X k k 2 k k
(d) = 1 + ( 1) + ( 1) + + ( 1)
1 2 k
djn

P
n n n k k n
Si recordamos el binomio de Newton, a b = (a + b) ; tenemos
k=o k
que
X k
(d) = (1 1) = 0:
djn

Particularizando el teorema 4.1.2.

Por ejemplo, si n = 24, luego todos los divisores positivos de 24 seran


f1; 2; 3; 4; 6; 8; 12; 24g, entonces

(1) = 1
(2) = 1
(3) = 1
22 = 0
(2 3) = 1
23 = 0
22 3 = 0
3
2 3 = 0

Finalmente,
X
(d) = (1) + (2) + (3) + 22 + (2 3) + 23 + 22 3 + 23 3
dj24
= 1 1 1+0+1+0+0+0
= 0

4.1.3 Denicin. La funcin indicatriz de Euler ' (n) : Si n 1 la indi-


catriz de Euler es el nmero de enteros positivos menores que n que son
coprimos con n; as
n
X
' (n) = 1; donde (k; n) = 1
k=1

68
Por ejemplo; si n = 12; entonces, se tiene que: (1; 12) = 1; (5; 12) = 1;
(7; 12) = 1; (11; 12) = 1 y luego,

' (12) = 1 + 1 + 1 + 1 = 4
En la siguiente tabla se muestran algunos valores para ' (n) ;

n: 1 2 3 4 5 6 7 8 9 10
' (n) : 1 1 2 2 4 2 6 2 6 4

4.1.3 Teorema. Para n 1 tenemos


X n
' (n) = (d)
d
djn

Demostracin. La suma que dene a ' (n) se puede escribir en la forma


n
X n
X 1
' (n) = 1 = ' (n) = ;
(n; k)
k=1 k=1

en donde k recorre todos los enteros k n: Ahora utilizaremos el teorema


4.1.2 sustituyendo n por (n; k) y obtenemos
0 1
Xn X n X
X
' (n) = @ (d)A = (d)
k=1 dj(n;k) k=1 djn
djk

Para un divisor d de n jo podemos sumar respecto de todos los k tales que


1 k n son mltiplos de d: Si escribimos k = qd entonces 1 k n si, y
slo si, 1 qd djn: Por lo tanto la ltima suma que da ' (n) se puede escribir
n=d
n X n=d
X X X X n
' (n) = (d) = (d) 1= (d) :
d
k=1 q=1 djn q=1 djn

Lo que demuestra el teorema.

Acontinuacin tenemos una expresin que conecta a ' (n) y a los divisores
primos de n:

4.1.4 Teorema. Para n 1 tenemos


Y 1
' (n) = n 1
p
pjn

69
Demostracin. Para el caso n = 1 el producto es vaco puesto que no
existe ningn primo que divida a 1, en este caso podemos indicar ' (1) = 1:
Suponemos, entonces, que n > 1 y que p1 ; p2 ; : : : ; pr son los divisores primos
distintos de n. El producto se puede escribir

Y r
Y
1 1
1 = 1
p i=1
pi
pjn

Observemos con ms detalle este producto, considrese los primeros tres


productos, esto es

Q
3
1 1 1 1
1 pi = 1 p1 1 p2 1 p3
i=1

p1 1 p2 1 p3 1
= p1 p2 p3

p1 +p2 +p3 p1 p2 p1 p3 p2 p3 +p1 p2 p3 1


= p1 p2 p3

p1 p2 p3 p1 p2 p1 p3 p2 p3 p1 p2 p3 1
= p1 p2 p3 + p1 p2 p3 + p1 p2 p3 p1 p2 p3 p1 p2 p3 p1 p2 p3 + p1 p2 p3 p1 p2 p3

1 1 1 1 1 1 1
= p1 p2 p2 p3 p1 + p1 p3 + p2 p3 p1 p2 p3 +1

1 1 1 1 1 1 1
= 1 p1 p2 p3 + p1 p2 + p1 p3 + p2 p3 p1 p2 p3

P 1
P 1 1
= 1 pi + pi pj p1 p2 p3

Si extendemos este producto hasta i = r encontramos que


r
Y X 1 X 1 X r
1 1 ( 1)
1 =1 + + + ( )
i=1
pi pi p i pj p i pj pk p1 p2 pr

P En 1
la parte derecha de sta ltima expresin vemos que un trmino como
pi pj se consideran todos los posibles productos pi pj de factores primos dis-
tintos de n; lo mismo ocurre para los casos pi pj pk ; pi pj pk pl;
Obsrvese que cada uno de los trminos de la derecha en ( ) es de la forma
1
d ; en donde d es un divisor de n que es 1 o producto de primos distintos. El
numerador 1 es claramente (d) : As podemos escribir entonces
r
Y X 1 X 1 X 1 (d) X (d)
r
1 ( 1) (d)
1 =1 + + + = + + +
i=1
pi pi pi p j p i pj pk p1 p 2 pr 1 di dr

70
P (d)
lo cual nos conduce exactamente a d ; Finalmente incluyendo n en el pro-
djn
ducto y considerando el teorema 4.1.3 resulta
r
Y X
1 n
n 1 = (d) = ' (n)
i=1
pi d
djn

que es lo que queriamos demostrar.

A manera de ejemplo, consideremos n = 60 y encontremos el nmero de


coprimos que posee.
Esto se reduce a calcular ' (60) ; el teorema nos da un buen algoritmo para
hacerlo, adems 60 = 22 3 5
3
Y 1 1 1 1
' (60) = 60 1 = 60 1 1 1 = 16
i=1
pi 2 3 5

La indicatriz de Euler, es una de las funciones ms importantes en teora de


nmeros por su alcance en muchos resultados notables, esto en gran medida por
sus propiedades; resumimos algunas en el siguiente teorema.

4.1.5 Teorema. La indicartiz de Euler cumple con las propiedades siguientes:

1
a. ' (p ) = p p para p primo y >1
d
b. ' (mn) = ' (m) ' (n) '(d) ; donde d = (m; n)

c. Si mjn entonces ' (m) j' (n)


d. ' (n) es par para n 3: Adems, si n posee r factores primos impares
distintos, entonces 2r j' (n)

Demostracin. La parte (a) se obtiene si en ( ) hacemos n = p ; as


Y 1
' (p ) = p 1
p
pjp

Luego, el nico primo p que divide a p es el mismo p; de donde

1 1
' (p ) = p 1 =p p
p

71
Por ejemplo;

' 73 = 73 72 = 343 49 = 294

y por otro lado, 73 = 343


Y 1 1
' (343) = 343 1 = 343 1 = 294:
p 7
pj343

Para probar la parte (b) escribimos

' (n) Y 1
= 1
n p
pjn

Teniendo presente que cada divisor primo de mn es un divisor primo de m o


n; y aquellos divisores primos que dividen tanto a m como a n dividen tambin
a (m; n) : De aqu que
Q 1
Q 1
1 p 1 p
' (mn) Y 1 pjm pjn
'(m) '(n)
m n
= 1 = Q = '(d)
mn p 1 1
pjmn p d
pj(m;n)

y nalmente
' (m) ' (n) d d
' (mn) = mn = ' (m) ' (n)
m n ' (d) ' (d)
Por ejemplo;
3 Y 1 Y 1 3
' (9 12) = ' (9) ' (12) =9 1 12 1 Q
' (3) p p 3 1 1
pj9 pj12 p
pj3
1
= 6 4 2 = 36
3

y por otro lado, 9 12 = 108


Y 1 1 1
' (108) = 108 1 = 108 1 1 = 36:
p 2 3
pj108

A partir de lo probado en (b) deducimos (c) : Si mjn tenemos n = mk en


donde 1 k n:Si k = n; entonces m = 1 y la parte (c) se verica trivialmente.
Por consiguiente, suponemos que c < b: De la parte (b) tenemos

d ' (k)
' (n) = ' (mk) = ' (m) ' (k) = d' (m) (y)
' (d) ' (d)

72
en donde d = (m; k) : Ahora el resultado se obtiene por induccin sobre n:
Para n = 1 se sigue trivialmente. Suponemos, pues, que (c) se verica para todo
entero menor que n:
Entonces se verica para k luego, puesto que djc; ' (d) j' (c) : Por lo que el
segundo miembro de (y) es un mltiplo de ' (m) ; es decir, ' (m) j' (n) : Lo que
prueba (c) :

Por ejemplo, 4j36 y ' (4) = 2; ' (36) = 12; de donde es claro que ' (4) j' (36) :

Ahora probaremos la parte (d) : Si n = 2 , 2 la parte (a) prueba


que ' (n) es par. Para el caso n = 3 obtenemos, ' (3) = 3 1 31 = 2; par.
As podemos establecer que ' (n) es par, para algn n 3; y probaremos que
tambin es cierto para n + 1; es decir que ' (n + 1) es par. Escribamos, entonces
Q
(p 1)
Y 1 pj(n+1) (n + 1) Y
' (n + 1) = (n + 1) 1 = (n + 1) Q = Q (p 1)
p p p
pj(n+1) pj(n+1)
pj(n+1) pj(n+1)

observando el factor (p 1) ; notamos que debe ser par, puesto que el nico
primo par es 2 y todo nmero impar es de la forma 2k + 1; de aqu que ' (n + 1)
sea par. Adems, cada primo impar proporciona una factor 2 a este producto,
por lo tanto 2r j' (n) ; si n tiene r factores primos impares distintos.

Por ejemplo,
1 1 1
' (30) = 30 1 2 1 3 1 5 = 8 = 23 ; es decir 23 j' (30)

73
4.1 Ejercicios Resueltos sobre Funciones Aritmticas

1. Cul es la mayor potencia de 2 que divide a 533!? La mayor potencia de


3?

533
Solucin. 2 = 266; 266
2 = 133; 133
2 = 66; 66
2 = 33;
33
2 = 16;
16 8 4 2
2 = 8; 2 = 4; 2 = 2; 2 = 1: Sumando se encuentra que 2529 j533!

533 177 59 19 6
Para 3; tenemos 3 = 177; 3 = 59; 3 = 19; 3 = 6; 3 = 2;
2
3 = 0:

Sumando estos resultados, encontramos 3263 j533!:

2. Si se escribe 100! en la notacin decimal ordinaria, sin el signo factorial,


cuntos ceros se escribiran en lnea en el extremo derecho?

Solucin. Es suciente con calcular la mayor potencia de 10 que divide a


100: Puesto que 10 = 2 5, tendremos que calcular la mayor potencia de 2 y
la mayor potencia de 5 que dividen a 100: De estos habr que tomar el menor
(porqu?), as

100 100 100 100 100 100


+ + + + + = 97
2 4 8 16 32 64
y
100 100
+ = 24
5 25
As, 100! tiene 24 ceros.

1
3. Para cualquier nmero real x probar que [x] + x + 2 = [2x]

Solucin. Escribamos x = m + "; con m 2 Z; luego [x] = m:


[m + "] + m + " + 21 = m + ["] + m + " + 12 = 2m + ["] + " + 21 : Con-
sideremos 0 " < 12 ; entonces, ["] = 0 y " + 12 = 0; por tanto se tiene que

1
[x] + x + = 2m = [2x]
2
1
Ahora consideremos 2 " < 1; entonces, 1 2" < 2; y de aqu que
" + 12 = 1:
Escribamos
1 1
[m + "] + m + " + = 2m + " + " + = 2m + 1
2 2

74
y
[2x] = [2 (m + ")] = 2m + [2"] = 2m + 1
por lo tanto
1
[x] + x + = 2m + 1 = [2x]
2

4. Para nmeros reales positivos cualesquiera x y y probar que [x] [y] [xy]

Solucin. Escribamos x = m + "; y = n + ; con m; n 2 Z; luego [x] = m y


[y] = n:
[m + "] [n + ] = (m + ["]) (n + [ ]) = mn + m [ ] + n ["] + ["] [ ] mn +
m + n" + " = [xy] :

5. Encontrar un entero positivo n tal que (n) + (n + 1) + (n + 2) = 3:

Solucin. n = 33:
2 2
(33) + (34) + (35) = (3 11) + (2 17) + (5 7) = ( 1) + ( 1) +
2
( 1) = 3

6. Probar que (n) (n + 1) (n + 2) (n + 3) = 0, si n es un entero positivo.

Solucin. Para n = 1; tenemos


(1) (2) (3) (4) = (1) ( 1) ( 1) (0) = 0
Puesto que los nmeros n; n + 1; n + 2 y n + 3 son cuatro nmeros enteros
consecutivos, siempre uno de ellos es de la forma 2r k; con r 2: Por lo tanto,
uno de estos nmeros admite siempre una descomposicin del tipo 2r p2 2 pk k ;
r
luego (2 p2 2
pk ) = 0; y de aqu que, (n) (n + 1) (n + 2) (n + 3) =
k

0:

n
7. Buscar todos los enteros n tales que ' (n) = 2

Solucin. Todos los nmeros de la forma 2r con r 1: Escribamos n = 2r


y r = 1; tenemos
2
' (2) = 1 =
2
as aceptamos la verdad de
2r
' (2r ) =
2
Probaremos que es cierto para r + 1
2r 2r+1
' 2r+1 = ' (2r 2) = ' (2r ) ' (2) (2) = 1 2= :
2 2

75
8. Calcular ' (n) para n = 64; 128

Solucin. 64 = 26 y 128 = 27 , as por el ejercicio anterior tenemos


64
' (64) = = 32
2
128
' (128) = = 64
2

9. Demostrar que ' (nm) = n' (m) si todo primo que divide a n tambin divide
a m:

Solucin. Observe que todos los primos que guran en el producto nm;
tambin aparecen en la descomposicin cannica de m y n:
" # " #
Q 1
Q 1
Q 1
' (nm) = nm 1 p = n m 1 p = n m 1 p =
pjnm pjnm pjm
n' (m) :

10. Supongamos m > 1. Probar que '(m) = m 1 s y solo si m es primo.

Solucin. Lo anterior se traduce como

' (m) = m 1 () m es primo

(=)) Si ' (m) = m 1; entonces signica que los nmeros 1; 2; 3; : : : ; m 1


son todos coprimos con m; de donde se deduce que el nico divisor menor que
m es 1 y por lo tanto m es primo.
Q
((=) Si m es primo, entonces ' (m) = m 1 p1 = m 1 m 1
= m 1:
pjm

76
4.2 Ejercicios Propuestos sobre Funciones Aritmticas.

1. Para qu nmeros reales x es verdad que


a. [x] + [x] = [2x] ;
b. [x + 3] = 3 + [x] ;
c. [x + 3] = 3 + x;
1 1
d. x + 2 + x 2 = [2x] ;
e. [9x] = 9 ?
2. Si n es cualquier entero positivo y cualquier nmero real, prubese que

1 n 1
[ ]+ + + + + = [n ]
n n

3. Encuntrese la mayor potencia de 13 que gura en 1729!


4. Calclese ' (666) ; ' (153) ; ' (16 384)
5. Demustrese que ' (mn) = ' (m) ' (n) ; si (m; n) = 1
6. Encuntrese todos los enteros n tales que

' (n) = ' (2n) ; ' (n) = 24

7. Caractercese el conjunto de enteros positivos que satisfacen ' (2n) > ' (n)
P
8. Calclese ' (d)
dj1729

9. Para cada proposicin establzcase una demostracin o presentar un con-


traejemplo
9.1 Si (m; n) = 1 entonces (' (m) ; ' (n)) = 1
9.2 Si n es compuesto, entonces (n; ' (n)) > 1:

77
5 El Anillo de los Enteros Mdulo n

Los encantos de esta ciencia sublime, las matemticas, slo se le revelan a


aquellos que tienen el valor de profundizar en ella.
Carl Friedrich Gauss

De aqu en adelante trataremos la aritmtica desde una nueva perspectiva,


la aritmtica modular. Introduciremos la congruencia mdulo n, las ecuaciones
de congruencias, los sistems de congruencias y dems. Estudiaremos tambin
en este captulo los resultados sobre congruencias ms importantes, el teorema
chino del resto y el pequeo teorema de Fermat y sus aplicaciones prcticas.

5.1 Generalidades sobre Congruencias mdulo n

Consideremos cuatro cajas y repartamos los nmeros enteros en cada una de


ellas de manera ordenada como sigue:
.. .. .. ..
. . . .
8 7 6 5
4 3 2 1
0 1 2 3
4 5 6 7
8 9 10 11
16 17 18 19
.. .. .. ..
. . . .

0 1 2 3

Las cajas las rotularemos as: 0 por contener al 0; ( o bien 0 + 4Z; esto es
lo mltiplos de 4); 1 por contener al 1( o bien 1 + 4Z; esto es lo mltiplos de
4 ms 1); 2 por contener al 2 ( o bien 2 + 4Z; esto es lo mltiplos de 4 ms
2); y la caja 3 por contener al 3 ( o bien 3 + 4Z; esto es lo mltiplos de 4 ms
3): Consideremos el conjunto Z4 = 0; 1; 2; 3 llamado conjunto completo de
residuos mdulo 4; pues al dividir cualquier entero entre 4 da residuos 0; 1; 2;
3:

5.1.1 Denicin. Dos enteros son congruentes mdulo n; lo que escribimos


a b mod n:
a b mod n () nja b
a b mod n () [a]n = [b]n
a b mod n () ra = rb

78
En caso contrario, decimos que a y b son incongruentes mdulo n.

Hemos dado en realidad tres deniciones sobre congruencia mdulo n; la


ventaja radica en el contexto de su uso, segn convenga podemos usar una
u otra denicin. En la primera lo hacemos en virtud de la divisibilidad, en
la segunda decimos que dos nmeros son congruentes, si las clases a las que
pertenecen son iguales, y nalmente decimos que dos nmeros son congruentes
si dejan el mismo resto en la divisin por n:

5.1.2 Denicin. Una relacin en un conjunto S se llama relacin de


equivalencia si satisface
a. a a (reexividad)
b. a b implica que b a (simetra)
c. a b^b c implica que a c (transitividad); 8a; b; c 2 S

5.1.1 Teorema. La congruencia mdulo n es una relacin de equivalencia.

Demostracin.
Sabemos que nj0, entonces nja a; luego a a mod n (ref lexividad) :
Si a b mod n; tenemos que nja b, entonces nj (a b) ; luego njb a que
es equivalente a, b a mod n (simetr{a) :

Si a b mod n; entonces nja b; si b c mod n; entonces njb c; luego


nj (a b) + (b c) de aqu que nja c que es equivalente a, a c mod n (tran-
sitividad).

5.1.3 Denicin. Clase de equivalencia del elemento a 2 S es el conjunto de


todos los elementos de S equivalentes a a:

ka = fx 2 S : x ag

Ntese que de la denicin anterior tenemos que: la unin de todas las clases
de equivalencias coincide con todo S y dadas dos clases distintas stas siempre
son disjuntas.
El conjunto formado por las clases de equivqlencias se llama conjunto co-
ciente de S por la relacin de equivalencia y lo escribimos as
S
= fki : i 2 Ig

79
Dado que la relacin de equivalencia que nos ocupa es la congruencia, vamos
a caracterizar las clases de equivalencia que esta forma.

A la clase ka pertenecen todos los enteros del tipo a = b + nk; donde b y n


estn dados, y k recorre todo Z: Luego decimos que Z queda particionado en
clases de equivalencias.
Por ejemplo
k0 = f: : : ; 2n; n; 0; n; 2n; : : :g

k1 = f: : : ; 1 2n; 1 n; 1; 1 + n; 1 + 2n; : : :g

k2 = f: : : ; 2 2n; 2 n; 2; 2 + n; 2 + 2n; : : :g
Como los subndices de cada clase son los posibles restos de la divisin por
n; las llamamos clases de restos mdulo n y se estila escribirlas as

0; 1; 2; 3; 4 : : :

y el conjunto cociente

Zn = 0; 1; 2; 3; 4 : : : ; n 1

5.1.4 Denicin. es compatible con () a a0 ^ b b0 =) a b a0 b0


cualesquiera que se a; b; a0 ; b0 2 S:

5.1.1 Proposicin. La congruencia mdulo n es compatible con la adicin y


la multiplicacin en Z: Es decir
a. a a0 mod n ^ b b0 mod n =) a + b a0 + b0 mod n
b. a a0 mod n ^ b b0 mod n =) a b a0 b0 mod n

Demostracin.
a. a a0 mod n =) a a0 = nk def. congruencia y divisibilidad
b b0 mod n =) b b0 = nk 0 def. congruencia y divisibilidad
=) (a + b) (a0 + b0 ) = nk 00 sumando ambas igualdades
=) nj (a + b) (a0 + b0 ) def. divisibilidad
=) (a + b) (a0 + b0 ) mod n def. congruencia

b. a a0 mod n =) a a0 = nk (1) def. congruencia y divisibilidad


b b0 mod n =) b b0 = nk 0 (2) def. congruencia y divisibilidad
=) ab a0 b = n (kb) multiplicando por b (1)
=) a0 b a0 b0 = n (a0 k 0 ) multiplicando por a0 (2)
=) ab a0 b0 = nk " sumando las dos ltimas igualdades
=) ab a0 b0 mod n: def. de congruencias.

80
Este resultado es de notable importancia, que una relacin de equivalencia
sea compatible con una ley de composicin interna denida en un conjunto,
nos induce una ley de composicin interna en el conjunto cociente, esto es, nos
permite operar con clases de equivalencias.

Vamos a denir en Zn la suma y el producto de clases en la forma siguiente:

a+b = a+b
a b = a b

Tambin se estila la notacin

[a]n + [b]n = [a + b]n


[a]n [b]n = [a b]n

Para nes prcticos, dadas dos clases, se suman o se multiplican (segn sea
el caso) sus representantes y la suma obtenida (o producto) se divide por el
mdulo propuesto y se admite como resultado de la operacin el resto de la
divisin.

As, con estas operaciones la tripleta (Zn ; +; ) forma un anillo, el anillo de


enteros mdulo n:

Construiremos la tabla para Z4 de la suma y el producto.

Z4 = 0; 1; 2; 3

+ 0 1 2 3 0 1 2 3
0 0 1 2 3 0 0 0 0 0
1 1 2 3 0 1 0 1 2 3
2 2 3 0 1 2 0 2 0 2
3 3 0 1 2 3 0 3 2 1

Un resultado importante es el hecho que (Zn ; +; ) es campo cuando n es


primo, lo cual expresamos con mayor formalidad en el siguiente enunciado.

5.1.2 Proposicin. Zp es cuerpo si, y solo si, p es un nmero primo.

Zp es cuerpo () p es primo

Demostracin.

i. (=)) Zp es un cuerpo, entonces p es nmero primo.

81
Supongamos que p no es un nmero primo; entonces existen dos nmeros
naturales q y t distintos de 1 y de n tales que p = q t: Por tanto
[q]p [t]p = [q t]p = [p]p = [0]p
y siendo [q]p y [t]p dos clases de nmeros congruentes no nulas, por tanto Zp
tiene divisores de cero, luego no es un cuerpo, esto contradice nuestra hiptesis.
En consecuencia p debe ser primo.

ii. ((=) p es un nmero primo, entonces Zp es un cuerpo.

Zp es un anillo unitario y conmutativo y para mostrar que es un cuerpo se


tiene que ver que todo elemento no nulo posee inverso.
Sea [m]n un elemento no nulo de Zp ; es evidente que m es menor que n y
que siendo n primo, m y n son primos entre s.
Por el torema 2.2.3 existen otros dos nmeros enteros a y b tales que
am + bn = 1
Por tanto, sus clases verican
[a]n [m]n + [b]n [n]n = 1
Pero [n]n = [0]n se obtiene,
[a]n [m]n = 1
luego, [m]n posee inverso y en consecuencia Zp es cuerpo.

Ejemplo.
Construiremos las tablas para la suma y el producto de Z5 ; con el n de
evidenciar que todo elemento no nulo es inversible.
Z5 = 0; 1; 2; 3; 4
+ 0 1 2 3 4 0 1 2 3 4
0 0 1 2 3 4 0 0 0 0 0 0
1 1 2 3 4 0 1 0 1 2 3 4
2 2 3 4 0 1 2 0 2 4 1 3
3 3 4 0 1 2 3 0 3 1 4 2
4 4 0 1 2 3 4 0 4 3 2 1
Observando con detenimiento resulta claro que

1 1 = 1
2 3 = 1
3 2 = 1
4 4 = 1
Todo elemento no nulo de Z5 es inversible, luego Z5 es cuerpo.

82
5.1.2 Teorema. Si a b mod n y mod n; entonces tenemos;
a. ax + y bx + y mod n
b. a b mod n
c. am bm mod n; 8a; b; x; y 2 Z

Demostracin. Si a b mod n; entonces, nja b; adems mod n


entonces nj ; eventualmente njx (a b) ^ njy ( ) luego n dividir la
suma, esto es njx (a b) + y ( ) ; reescribiendo este trmino nj (ax + y)
(bx + y) ; que por denicin de congruencia podemos escribir ax + y bx +
y mod n y queda probada la parte (a) :

Para la parte (b) tenemos, si a b mod n; entonces, nja b; adems


mod n entonces nj ;eventualmente nj (a b) ^ njb ( ) luego n di-
vidir la suma, esto es nj (a b) + b ( ) ; de donde es claro que nja b ;
que por denicin de congruencia es a b mod n:

Finalmente para la parte (c) tenemos que am bm mod n; es verdadero


para m = 1: Luego por la parte (b) podemos escribir am a bm b mod n o
m+1 m+1
equivalentemente a b mod n con lo que queda probada la parte (c) y
el teorema.

Criterios de divisibilidad y la congruencia mdulo n.

Ejemplo 1. Divisibilidad por 9:


Un entero n > 0 es divisible por 9 si, y slo si, la suma de los dgitos
de su expresin decimal es divisible por 9: Probaremos este hecho usando la
congruencia y sus propiedades ya enunciadas. Escribiendo n en su expresin
decimal tenemos
n = a0 + 10a1 + 102 a2 + + 10k ak
Por el teorema anterior tenemos que
9
1 1 =) a0 a0 >
>
>
>
10 1 =) 10a1 a1 >
=
102 1 =) 102 a2 a2 mod 9
.. .. .. >
>
. . . >
>
>
;
10k 1 =) 10k ak ak

luego tenemos que

a0 + 10a1 + 102 a2 + + 10k ak (a0 + a1 + a2 + + ak ) mod 9

lo que equivale a escribir

n a0 + a1 + a2 + + ak mod 9

83
Particularmente, tenemos n = 1305; luego 1 + 3 + 0 + 5 = 9 por tanto 9j1305:

Ejemplo 2. Divisibilidad por 17:


Un entero n > 0 es divisible por 17 si, y slo si, al quitar sus dos ltimas
cifras y restarlas del duplo de lo que queda, el resultado es mltiplo de 17:

Sin prdida de generalidad, consideremos un nmero n de 4 cifras.

Escribamos n = 1000a3 + 100a2 + 10a1 + a0 ; luego n = 100 (10a3 + a2 ) +


(10a1 + a0 ) :

Hagamos,
n0 = (10a3 + a2 ) y a = (10a1 + a0 )
.

Podemos escribir n en la forma n = 100n0 + a; n0 sern la cifras que queda


al retirar las dos ltimas, esto es al quitar a:

As, lo que debemos probar es

17jn =) 2n0 a 0 mod 17

Si 17jn, entonces 100n0 + a 0 mod 17; luego 100n0 + a 15n0 + a mod 17 y


15n0 + a 2n0 + a mod 17:

A partir de esto podemos plantear que, 100n0 + a 0 mod 17 y 100n0 + a


2n0 + a mod 17:

De donde resulta (transitividad de la congruencia) 0 2n0 + a mod 17


0
equivalentemente 2n a 0 mod 17; que es el resultado que buscabamos.

Ejemplo.
Particularmente, tenemos n = 4267; hacemos n0 = 42 y a = 67; luego
2 (42) 67 = 17 por tanto 17j4267:

5.1.3 Teorema. Si c > 0 entonces

a b mod n si, y slo si, ac bc mod nc

Demostracin. Tenemos nj (a b) si, y slo si (a b) = nk, multiplicando


por c ambos lados de esta igualdad resulta (a b) c = nck; que podemos traducir
en ncj (a b) c y nalmente ac bc mod nc:

84
5.1.4 Teorema. Ley de simplicacin. Si ac bc mod n y si d = (n; c) ;
entonces

n
a b mod
d

Demostracin. Dado que ac bc mod n; tenemos nj (a b) c esto es


(a b) c = nk; luego como d = (n; c) es posible escribir (a b) dc = nd k; lo
que traducimos en nd j (a b) dc , pero nd - dc puesto que nd ; dc = 1; luego debe
ser que nd j (a b) o equivalentemente a b mod nd :

5.1.5 Teorema. Suponemos que a b mod n: Si djn y dja, entonces djb:

Demostracin. Suponiendo d > 0:


a b mod n =) nja b
djn =) dja b
de las implicaciones anteriores se deduce que
a b mod d

Por hiptesis sabemos que dja; luego 0 a mod d ^ a b mod d (transi-


tividad de la congruencia ) se deduce que 0 b mod d; equivalentemente djb:

5.1.6 Teorema. Si a b mod n entonces (a; n) = (b; n) : En otras palabras,


los nmeros que son congruentes mdulo n tienen el mismo mcd con n:

Demostracin. Sea d = (a; n) y e = (b; n) : Entonces djn y dja; luego djb;


por lo tanto dje: Anlogamente, ejn; ejb; luego eja; por lo tanto ejd: Por lo tanto
debe ser d = e:

5.1.5 Denicin. Si x y mod n entonces y recibe el nombre de resto de x


mdulo n: Un conjunto fx1 ; x2 ; : : : ; xn g es un sistema completo de restos
mdulo n; si para todo entero y existe uno y solamente un xj tal que
y xj mod n:

Ejemplo.
Todo conjunto formado de n enteros, incongruentes mod n; es un sistema
completo de restos mod n.

As, f8; 9; 10; 11g es un sistema completo de resto mod 4; puesto que 8 2
0; 9 2 1; 10 2 2 y 11 2 3:

85
5.1.6 Denicin. Un sistema reducido de restos mdulo n es todo conjunto de
enteros fx1 ; x2 ; : : : ; xn g, incongruentes mdulo n; cada uno de ellos primos
con n:

Nota: ' (n) es el valor para n de la indicatriz de Euler, introducida en el


captulo 4.

Por ejemplo. ' (4) = 2 y f8; 9; 10; 11g es un sistema completo de resto
mod 4; luego f9; 11g es un sistema reducido de restos, note que 9; 11 son coprimos
con 4:

5.1.7 Teorema. Suponemos que (k; n) = 1: Si fa1 ; a2 ; a3 ; : : : ; an g es un sis-


tema completo de restos mdulo n; tambin lo es fka1 ; ka2 ; ka3 ; : : : ; kan g

Demostracin. Supongamos que kai kaj mod n entonces ai aj mod n


por el teorema 5.1.4 ya que (k; n) = 1; y esto contradice el hecho que fa1 ; a2 ; a3 ; : : : ; an g
sea un sistema completo de restos. Por tanto ningn par de elementos del con-
junto fka1 ; ka2 ; ka3 ; : : : ; kan g es congruente mod n: Puesto que en este conjunto
existen n elementos incongruentes, constituye un sistema completo de resto.

5.1.8 Teorema. Si a1 ; a2 ; a3 ; : : : ; a'(n) es un sistema residual reducido m-


dulo n y si (k; n) = 1; entonces ka1 ; ka2 ; ka3 ; : : : ; ka'(n) es tambin un
sistema residual reducido mdulo n:

Demostracin. Por lo dicho en el teorema anterior, sabemos que ningn


par de nmeros kai es congruente mdulo n: Adems, puesto que (ai ; n) =
(k; n) = 1, tenemos que (kai ; n) = 1; luego cada kai es primo con n:

Toda la discusin anterior ha sido necesaria para llegar a uno de los resulta-
dos ms importantes de la teora de congruencias. Los teoremas de Euler-Fermat

5.1.9 Teorema. Terorema de Euler. Suponemos que (a; n) = 1: Entonces


tenemos que
a'(n) 1 mod n

Demostracin. Sea b1 ; b2 ; b3 ; : : : ; b'(n) un sistema residual reducido m-


dulo n: Entonces ab1 ; ab2 ; ab3 ; : : : ; ab'(n) es tambin un sistema residual re-
ducido. Consideremos el producto b1 b2 b3 b'(n) y ms precisamente su
resto al dividir por n; note que coincide con el resto de ab1 ab2 ab3 ab'(n) :

86
Por lo tanto el producto de todos los enteros del primer conjunto es congruente
con el producto de los del segundo conjunto mod n. Por consiguiente

b1 b2 b'(n) ab1 ab2 ab'(n) mod n


'(n)
b1 b2 b'(n) a b1 b2 b'(n) mod n

Cada bi es coprimo con n; por lo tanto se pueden simplicar de ambos lados


y obtener nalmente
a'(n) 1 mod n:

5.1.10 Teorema. Pequeo Teorema de Fermat. Considrese que p


denota un primo. Si p - a; entonces

ap 1
1 mod p

Demostracin. Si p - a entonces (a; p) = 1 y a'(p) 1 mod p por el teorema


de Euler. Por el ejercicio 10 del captulo anterior tenemos que ' (p) = p 1 y
se tiene que
ap 1 1 mod p:

5.2 Congruencias Lineales

Nuestro objetivo en esta seccin es el estudio de las congruencias de la forma

f (x) 0 mod n (1)

Resolver la congruencia, signica hallar todos los valores de x que la satis-


facen. Dos congruencias, a las que satisfacen un mismo valor de x, se llaman
congruencias equivalentes.
Advirtase que si x es solucin de la ecuacin (1) tambin x + kn ser solu-
cin, as esta ecuacin tiene innitas soluciones. Por tanto convendremos en
considerar las soluciones tales que 0 x < n; que llamaremos soluciones prin-
cipales.

El caso de las congruencias lineales quedar totalmente descrita mediante


los tres teoremas siguientes.

5.2.1 Teorema. Si (a; n) = 1: Entonces la congruencia lineal

ax b mod n (2)

87
tienen exactamente una solucin.

Demostracin. Para la solucin principal debe considerarse nicamente


los nmeros 1; 2; 3; : : : ; n; adems estos nmeros forman un sistema completo
de restos. Por consiguiente formamos los productos a; 2a; : : : ; na: Puesto que
(a; n) = 1 estos nmeros constituyen un sistema residual completo. Por lo tanto,
slo uno de estos productos es congruente con b mdulo n.
Es decir, existe un nico x que satisface (2) :

5.2.2 Teorema. Suponemos que (a; n) = d. Entonces la congruencia lineal

ax b mod n (3)

tiene solucin si, y slo si, djb:

Demostracin. (=)) Si ax b mod n tiene solucin, entonces djb:


Si x es la solucin de la ecuacin (3) ; entonces kn = ax b; de donde
b = kn + ax: Si (a; n) = d; se tiene que dja, djn y eventualmente dividir a djax,
djkn: Por consiguiente tambin divide a la suma, esto es djkn + ax; por tanto
djb:

((=) Si djb entonces ax b mod n tiene solucin.


Si djb la congruencia
a b n
x mod (4)
d d d
a n
tiene una solucin por el teorema 5.2.1, puesto que d; d = 1; y esta solucin
es tambin una solucin de (3) :

5.2.3 Teorema. Suponemos que (a; n) = d y que djb: La congruencia lineal

ax b mod n

tiene exactamente d soluciones mdulo n: Vienen dadas por


n n n
t; t + ; t + 2 ; : : : ; t + (d 1) ; (5)
d d d
n
donde t es la solucin nica mdulo d; de la congruencia lineal
a b n
x mod (6)
d d d
Demostracin. Cada solucin de (6) es tambin solucin de (3) ; recpro-
camente cada solucin de (6) satisfase (3) : Basta considerar que
a b n a b n
x mod =) x = k =) ax b = kn =) ax b mod n
d d d d d d

88
Los d nmeros escritos en (5) son soluciones de (6) y, por lo tanto, de (5) :
Falta probar que (3) no tiene ms soluciones que las descritas en (5) : Si y es
una solucin de (3) entonces ay at mod n luego y t mod nd : Por lo tanto
n
y = t + k d para cierto k: Pero k r mod n para un r que verica 0 r < d:
Por consiguiente
n n n
k r mod n; luego y t + r mod n
d d d
Por consiguiente y es congruente mdulo n con uno de los nmeros descritos
en (5) : Esto termina la demostracin.

Por ejemplo. Encontrar las soluciones a la ecuacin

3x 5 mod 11

Inspeccionando el conjunto 0 x < 11, encontramos como solucin princi-


pal x = 9; puesto que 11j27 5: Luego el resto de soluciones tendrn la forma
x = 9 + 11k:

Ya hemos notado que la congruencia tiene mucha similitud con la igual-


dad, as que parece natural preguntarnos por los sistemas de ecuaciones de
congruencias y sus soluciones, esto es, dadas las ecuaciones a0 x b0 mod n y
a1 x b1 mod n; existen valores de x que las satisfacen a ambas? A este hecho
responde el siguiente teorema

5.2.3 Teorema. Teorema Chino del Resto. Supongamos que n1 ; n2 ; ; nr


son enteros positivos coprimos dos a dos:

(ni ; nk ) = 1 si i 6= k

Sean b1 ; b2 ; : : : ; br enteros arbitrarios. Entonces el sistema de congruencias


x b1 mod n1
x b2 mod n2
.. .. ..
. . .
x br mod nr
posee exactamente una solucin mdulo el producto n1 n2 nr

Demostracin. Escribiendo N = n1 n2 nr y Nk = nNk : Entonces


(Nk ; nk ) = 1; por lo tanto cada Nk admite un inverso nico Nk0 mdulo nk ;
esto es Nk Nk0 1 mod nk : Sea ahora

x = b1 N1 N10 + b1 N2 N20 + + br Nr Nr0


Dado que Ni 0 mod nk si i 6= k tenemos

x bk Nk Nk0 bk mod nk

89
Por lo tanto x satisface cada una de las congruencias del sistema. Adems el
sistema posee una nica solucin mdulo N: En efecto, si x y y son dos soluciones
del sistema tenemos x y mod nk para cada k y, puesto que los nk son dos a
dos coprimos, tendremos tambin x y mod N: Esto termina la demostracin.

5.3 Ejercicios Resueltos sobre Congruencias Lineales

1. Hacer una lista de todos los enteros x en el intervalo 1 x 100 que


satisfagan x 7 mod 17

Solucin. Si x 7 mod 17 entonces 17jx 7; luego x 7 = 17n y nalmente

x = 17n + 7

si evaluamos para n = 6, x toma el valor 109; que supera la condicin dada.


Entonces para obtener las soluciones pedidas recorremos n para 1; 2; 3; 4 y 5
obteniendo los valores buscados para x; que guran a continuacin

24; 41; 58; 75; 92

2. Mostrar un sistema completo de residuos mdulo 17 compuesto enteramente


de mltiplos de 3:

Solucin. Tomemos Z17 = 0; 1; 2; 3; 4;5; 6; 7; 8; 9; 10; 11; 12; 13; 14; 15; 16 ;
ahora basta escojer un elemento perteneciente a cada clase residual, con el
cuidado en la eleccin de ser mltiplo de 3:
Todos los enteros de 0 tienen la forma 0 + 17k; as 0 2 0 y adems 3j0; los
enteros de 1 tienen la forma 1 + 17k, luego 18 2 1 y 3j18; para la clase 2 los
enteros son de la forma 2 + 17k; luego 36 2 2:
Si seguimos este razonamiento y recordando que si dos nmeros dan el mismo
resto pertenecen a la misma clase; encontramos el siguiente sistema completo
de residuos mdulo 17

f0; 18; 36; 54; 21; 39; 6; 24; 42; 9; 27; 45; 12; 30; 48; 15; 33g

3. Los nmeros de Fermat. Pierre de Fermat arm que todo nmero de la


n
forma fn = 22 + 1 era primo. Probar que esta aseveracin es falsa.

90
Solucin. Los cinco primeros son primos:

f0 = 3; f1 = 5; f2 = 17; f3 = 257; y f4 = 65537

No as para f5 ; mostraremos que f5 es divisible por 641 sin calcular explci-


n
tamente f5 : Consideremos la sucesin de potencias 22 mdulo 641. Tenemos

22 = 4; 24 = 16; 28 = 256; 216 = 65536 154 mod 641;


luego
2
232 (154) = 23716 mod 641
23716 640 mod 641
640 1 mod 641
232 1 mod 641

Finalmente
f5 = 232 + 1 0 mod 641
lo que signica que f5 no es primo.

4. Encuentre el residuo cuando 1717 es dividido por 7:

Solucin.
Partamos del hecho que 17 3 mod 7; entonces 1717 317 mod 7; luego
2 4
tenemos que 3 = 9 2 mod 7; por tanto 3 4 mod 7: De ah que 38 16
16
2 mod 7 y 3 4 mod 7:

Por tanto
317 3 316 12 5 mod 7
Lo que nos conduce a

1717 317 5 mod 7

Luego el nmero buscado es 5:

5. Pruebe que 237 1 es un mltiplo de 223:

Solucin. 256 = 28 33 mod 223; entonces 216 332 mod 223 y 332
2 2
26 mod 223; por tanto 232 ( 26) mod 223 y ( 26) = 676 7 mod 223:
De ah
237 232 25 mod 223
232 25 7 32 mod 223
7 32 224 mod 223
224 1 mod 223

91
esto es
237 1 0 mod 223

6. Encuentre el resto de la divisin 1! + 2! + + 100! por 45:

Solucin. Sabemos que (h + 1)! = (h + 1) h! y que 6! = 720 = 16 45;


luego 45j6!: Por otra parte 1! + 2! + 3! + 4! + 5! = 153; observando con cuidado
153 18 = 135 = 3 45
Por todo lo anterior podemos escribir

1! + 2! + + 100! 1! + 2! + 3! + 4! + 5! 18 mod 45

7. Hallar los nmeros tales que, divididos por 2; 3; 4; 5 y 6 den como resto,
1; 2; 3; 4 y 5 respectivamente.

Solucin. De las condiciones del enunciado se obtienen las siguientes con-


gruencias, siendo x uno de tales nmeros:

x 1 mod 2 =) x+1 0 mod 2


x 2 mod 3 =) x+1 0 mod 3
x 3 mod 4 =) x+1 0 mod 4
x 4 mod 5 =) x+1 0 mod 5
x 5 mod 6 =) x+1 0 mod 6

Por tanto, x + 1 es mltiplo de 2; 3; 4; 5 y 6; es decir, x + 1 es mltiplo de


mcd(2; 3; 4; 5; 6) = 60: Luego, x = 60t 1; t 2 Z

8. Encuentre el residuo cuando 13 1245 es dividido por 47.

Solucin. Por el pequeo teorema de Fermat 1246 1 mod 47: Multipli-


cando cada lado por 4 tenemos; 4 1246 4 mod 47 lo que es equivalente a
(4 12) 1245 4 1 mod 47; entonces 1245 4 mod 47; multiplicando por 13;
tenemos

13 1245 52 mod 47

9. Pruebe que, Si ac bc mod n y (c; n) = 1; entonces a b mod n

Solucin. Si ac bc mod n; entonces njac bc esto es njc (a b) ; como n


es coprimo con c se tiene que n - c; entonces nj (a b) ; equivalentemente

a b mod n

92
10. Encontrar todas las soluciones para
a. 20x 4 mod 30
b. 42x 90 mod 156

Solucin. a. El mcd(20; 30) = 10 y 10 - 4; as por teorema 5.2.2 la ecuacin


20x 4 mod 30 no tiene solucin.
b. El mcd(42; 156) = 6 y 6j90; entonces existen 6 soluciones principales
incongruentes mdulo 156:Reducimos la congruencia dada a 7x 15 mod 26:
Luego se tienen las siguientes relaciones 33x 7x mod 26; entonces 33x
15 mod 26; esto ltimo puede escribirse como 11 3x 5 3 mod 26; como
mcd(3; 26) = 1 por el ejercicio anterior es posible escribir 11x 5 mod 26;
pero 15x 11x mod 26; tenemos por tanto 15x 5 mod 26 y dividiendo por
5, resulta 3x 1 27 mod 26 ahora dividiendo por 3; x 9 mod 26:
Finalmente
x 9 17 mod 26
Por lo tanto, 7x 15 mod 26 tiene una nica solucin x 17 mod 26: Por el
teorema 5.2.3 las soluciones buscadas son 17; 17 + 156 312 468 624
6 ; 17 + 6 ; 17 + 6 ; 7 + 6
780
y 7+ 6 :

11. Encuentre la nica solucin de 251x 125 mod 521. (521 es un primo).

521
Solucin. El entero ms cercano a 251 es 2, multiplicando por 2 la con-
gruencia y reduciendo a mdulo 521 obtenemos, 502x 250 mod 521: 19x
502x 250; entonces 19x 250 mod 521: Anlogamente multiplicamos por
27; el entero ms cercano a 521
19 ; obtenemos 513x 6750 mod 521; luego
8x 513x mod 521 y 6750 498 mod 521; as tenemos 8x 498 mod 521:
Una vez ms el entero ms cercano a 5218 es 65; multiplicando la congruencia
resulta, 520x 32370 mod 521; luego 32370 453 mod 521; esto conduce a
520x 453 mod 521:
Y nalmente la congruencia
x 520x 453 mod 521:
de donde la nica solucin a la congruencia original es x = 453:

12. Considere el sistema x a mod m, x b mod n donde m; n no son nece-


sariamente primos relativos. Pruebe que si (m; n) divide a, b a, entonces
el sistema tiene una solucin.

Solucin. Sea d = (m; n), y suponemos djb a. Entonces existen enteros


u; v tales que mu + nv = b a: Sea x = a + mu; claramente, x a mod m. Pero
x = a + mu = a + (b a) nv = b nv, de donde resulta
x b mod n.

93
13. Encuentre todas las soluciones a los pares de congruencias 3x 7y
4 mod 19, 7x 3y 1mod19.

Solucin. Puesto que (7; 19) = 1, la primera congruencia es equivalente a la


congruencia 7 (3x 7y) 7 4 mod 19, es decir, 21x 49y 28 mod 19: Similar-
mente, la congruencia 7x 3y 1 mod 19 es equivalente a 21x 9y 3 mod 19:
Restando ambas congruencias obtenemos, 40y 25 mod 19, o equivalente-
mente 2y 6 mod 19: Esta es una solucin y = 3 mod 19 sustituyendo esta
en la primera congruencia, obtenemos 3x 2 mod 19, dando x 7 mod 19:
Entonces la solucin al sistema es x 7 mod 19; y 16 mod 19:

14. Encuentre un entero x, 0 < x < 140, que satisface la congruencia x


1 mod 4, 2x 3 mod 5, 4x 5 mod 7:

Solucin. Primero ponemos las congruencias en la forma x ai mod mi ,


entonces aplicamos el teorema chino del resto. La primera congruencia es
aproximada a est forma; para 2x 3 mod 5; multiplicamos cada lado por 3
y reducimos a mdulo 5 obteniendo x 4 mod 5; para 4x 5 mod 7, multi-
plicamos cada lado por 2 y reducimos a mdulo 7, obteniendo x 3 mod 7.
Ahora encontremos b1 ; b2 ; b3 , tales que 5 7b1 1 mod 4, 4 7b2 1 mod 5,
4 5b3 1 mod 7; es decir, b1 1 mod 4; 3b2 1 mod 5; b3 1 mod 7. Por
tanto nosotros podemos tomar b1 = 1, b2 = 2, b3 = 1. De aqu una solucin
es x = 35( 1)(1) + 28(2)(4) + 20( 1)(3) = 129: Puesto que 4 5 7 = 140, 129
es la nica posible solucin a este sistema que es menor que 140.

94
5.4 Dnde estn las congruencias?

El nmero ISBN (International Standar Book Number), es un cdigo de dgitos


vericador de gran utilidad. El primer grupo de nmeros indica el pas o el
idioma, el segundo grupo de dgitos designa la editorial, el tercer grupo es asig-
nado al libro por la casa editorial y el ltimo dgito es un factor de comprobacin
de errores.

Pero la determinacin de este ltimo dgito se hace de una manera muy


peculiar, no se asigna al producto bajo algn criterio exclusivo como los ante-
riores, este dgito que denominaremos x se calcula a partir de los restantes. Si
designamos los primeros 9 dgitos como x1 ; x2 ; x3 ; x4 ; x5 ; x6 ; x7 ; x8 ; x9 el dcimo
dgito verica la relacin:
9
X
x10 ixi mod 11
i=1

Entonces para el caso de la gura tenemos

x10 (1 8) + (2 4) + (3 6) + (4 0) + (5 4) + (6 8) + (7 9) + (8 5) + (9 8) mod 11
x10 8 + 8 + 7 + 0 + 9 + 4 + 8 + 7 + 6 mod 11
x10 57 mod 11

Esto nos obliga a escoger a x10 = 2 como se muestra en la gura.

El cdigo ISBN detectar todos los errores simples y de transposicin, ac-


tualmente cualquier programa profesional que trabaje con el ISBN utiliza las
congruencias para determinarlo. El ordenador comprueba si el nmero intro-
ducido por el usuario con el escner coincide con el calculado por el [ordenador]
mismo, si esto no es as entonces existe algn error.

95
5.5 Ejercicios Propuestos

1. Escrbase las tablas de adicin y mltiplicacain para Z7 y Z8 :


2. Encuntrese y ennciese un crterio de divisibilidad para 3; 5 y 7 (para cada
uno)
3. Escrbase una sola congruencia que sea equivalente al par de congruencias
x 1 mod 4; x 2 mod 3
4. Prubese que si p es un primo y a2 b2 mod p; entonces pj (a + b) o bien
pj (a b) :
p
5. Demustrese la igualdad del estudiante. Si p es primo, entonces (a + b)
(ap + bp ) mod p:
77
6. Hllese las ltimas dos cifras del nmero 77
7. Calclese el residuo que se obtiene al dividir 7077377 entre 11:
8. Mustrese un sistema reducido de residuos mdulo 7 compuestos enteramente
por potencias de 3.
9. Prubese que n6 1 es divisible entre 7 si (n; 7) = 1
10. Prubese que n7 1 es divisible entre 42, para cualquier entero n:
11. Encuntrese el menor residuo positivo de :
11.a 3500 Mdulo 13
11.b 12! Mdulo 13
11.c 516 Mdulo 17
11.d 5500 Mdulo 17
12. Encuntrese todas las soluciones de
12.a 87x 57 mod 105
12.b 64x 897 mod 1001
12.c 36 x 1 mod 8180
13. Prubese. Teorema de Wilson. Si p es un primo, entonces (p 1)!
1 mod p
14. Encuntrese todas las soluciones del par de congruencias 3x 7y 4 mod
15, 7x 3y 1 mod 15
15. Encuntrese el menor entero positivo tal que x 5 mod 12, x 17 mod 20,
y x 23 mod 42

96
16. sese el teorema de Fermat para resolver la congruencia x35 +5x19 +11x3
0 mod 17
17. Para cualquier primo p; si ap bp mod p; probar que ap bp mod p2
18. Demustrese que la suma de los cuadrados de cinco nmeros enteros con-
secutivos, no puede ser nunca un cuadrado perfecto.
19. La suma de las cifras de un nmero es 20. Si de ese nmero se resta 205 y se
divide la diferencia por 2 se optiene por resultado el nmero formado por
las cifras del primero escritas en orden inverso. Encuntrese el nmero.
20. Prubese que la ecuacin 3x2 + 2 = y 2 no tiene solucin entera.

97
6 El ltimo Teorema De Fermat

Es dudoso que el ingenio humano pueda llegar a construir un enigma que el


propio ingenio humano no sea capaz de resolver.
Edgar Allan Poe.

En esta seccin nos ocuparemos del conocido ltimo teorema de Fermat. El


ltimo teorema de Fermat arma que la ecuacin xn + y n = z n ; donde n es un
entero mayor que 2, no tiene soluciones enteras, con la excepcin, las soluciones
triviales en las que una de las variables es 0. Aqu abordaremos este caso slo
en su aspecto ms elemental y para exponentes 4:

6.1 Nmeros Complejos C

No existe un nmero real x que satisfaga la ecuacin polinmica x2 +1 = 0. Para


resolver este tipo de ecuaciones, es necesario introducir los nmeros complejos.

7.1.1 Denicin. Un nmero complejo z es un nmero que se expresa en la


forma
z = a + bi
p
donde a y b son nmeros reales e i = 1: Decimos que a es la parte real
de z y que b es la parte imaginaria, lo cual escribimos como

Re (z) = a y Im (z) = b

7.1.2 Denicin. Dos nmeros complejos son iguales si y slo si sus partes
reales son iguales y sus partes imaginarias son iguales.

Es decir, si
z = a + bi y w = c + di
y
z=w
entonces
a=cyb=d

7.1.3 Denicin. Operaciones en C: Sean z = (a + bi) y w = (c + di) ; en-


tonces

98
z+w = (a + bi) + (c + di) = (a + c) + (b + d) i (adicin)
z w = (a + bi) (c + di) = (ac bd) + (ad + bc) i (multiplicacin)

A partir de la denicin 7.1.3 es fcil vericar que la terna (C; +; ) es un


cuerpo.

7.1.4 Denicin. Complejo Conjugado. Conjugado de z = a + bi es el nmero


complejo z = a bi: El smbolo z se lee conjugado de z.

7.1.1 Teorema. Si z y w son nmeros complejos, entonces:


i. z + w = z + w
ii. z w = z w
z z
iii. Si w 6= 0; w = w

iv. Si z es real, z = z
v. z + z = 2 Re (z)

Demostracin.

i. Si z = a + bi y w = c + di; con a; b; c y d reales, tenemos z + w = (a + c) +


(b + d) i; luego z + w = (a + c) (b + d) i = a bi + c di = z + w:
ii. Si z = a + bi y w = c + di; con a; b; c y d reales, tenemos z w = (ac bd) +
(ad + bc) i; z w = (ac bd) (ad + bc) i = (a bi) (c di) = z w:
iii. Si w = c + di; con c y d reales no nulos simultneamente, tenemos

1 1 c di
= = 2 ;
w c + di c + d2
1 1 c + di 1
= = 2 =
w c + di c + d2 w

De ah,
z 1 1 1 z
=z =z =z =
w w w w w

iv. Si z = a + 0i; z es real, z = a 0i = z:


v. Si z = a + bi y z = a bi; entonces z + z = (a + a) + (b b) i = 2a + 0i = 2a:
Como a = Re (z) ; se tiene que, z + z = 2a = 2 Re (z) :

99
7.1.5 Denicin. Mdulo de un complejo es la raz cuadrada no negativa de
la suma de los cuadrados de las partes real e imaginaria.

Esto es p
jzj = a2 + b2

7.1.2 Teorema. Sea z; z 0 2 C; entonces


i. Re (z) jzj
ii. Im (z) jzj
2
iii. z z = jzj
iv. jzz 0 j = jzj jz 0 j
v. jz + z 0 j jzj + jz 0 j

6.2 Enteros de Gauss

Nos detendremos en esta seccin para adquirir algunas herramientas algebraicas


que nos permitan tratar nuestro problema principal con mucha ms facilidad.

Denotemos por Z [i] el conjunto de todos los nmeros complejos de la forma


a + bi; donde a y b son enteros.
Es decir, el conjunto

Z [i] = fa + bi : a; b 2 Zg

Bajo la adicin y multiplicacin habituales de los nmeros complejos, Z [i]


forma un dominio de integridad, llamado el dominio de los enteros gaussianos
o enteros de Gauss.

7.2.1 Denicin. Se llama anillo unitario a un anillo A cuyo producto tiene


elemento neutro en A = A f0g : Dicho elemento se denomina uno y se
representa por 1A simplemente 1; si no hay riesgo de confusin.

7.2.2 Denicin. Sea A un anillo unitario. Una unidad de A es un elemento


x 2 A que tiene inverso y 2 A respecto del producto:

100
xy = yx = 1
El conjunto de todas las unidades de A se representa por U (A) ; una propiedad
importante del producto de anillos es que se verica la propiedad

U (A B) = U (A) U (B)

Demostracin.
Empezaremos escribiendo esta propiedad en el lenguaje conjuntista

U (A B) = U (A) U (B)

Si y solo si,

U (A B) U (A) U (B) ^ U (A) U (B) U (A B)

i. U (A B) U (A) U (B) :

Tomemos (a; b) 2 U (A B) ; por denicin de unidad, existe (a0 ; b0 ) tal que,

(a; b) (a0 ; b0 ) = (aa0 ; bb0 ) = (1A ; 1B ) :

Por denicin de A B; se sigue que, aa0 2 A y bb0 2 B; pero el producto


aa = 1A , por tanto a 2 U (A) y bb0 = 1B ; luego b 2 U (B) ; de aqu deducimos
0

que (a; b) 2 U (A) U (B) :

ii. U (A) U (B) U (A B) :

Tomemos (a; b) 2 U (A) U (B) ; por denicin de A B; a 2 U (A) y

b 2 U (B) ; por denicin de unidad, existe a0 2 A y b0 2 B; tal que, aa0 = 1A y


bb0 = 1B :

Por tanto (a; b) (a0 ; b0 ) = (1A ; 1B ) ; de aqu que el par (a; b) es unidad de

A B; i:e; (a; b) 2 U (A B) :

Ejemplo.
Z2 = 0; 1 y Z3 = 0; 1; 2 ; luego Z2 Z3 = 0; 0 ; 0; 1 ; 0; 2 ; 1; 0 ; 1; 1 ; 1; 2

Formemos la tabla para la multiplicacin en Z2 Z3

101
0; 0 0; 1 0; 2 1; 0 1; 1 1; 2
0; 0 0; 0 0; 0 0; 0 0; 0 0; 0 0; 0
0; 1 0; 0 0; 1 0; 2 0; 0 0; 1 0; 2
0; 2 0; 0 0; 2 0; 1 0; 0 0; 2 0; 1
1; 0 0; 0 0; 0 0; 0 1; 0 1; 0 1; 0
1; 1 0; 0 0; 1 0; 2 1; 0 1; 1 1; 2
1; 2 0; 0 0; 2 0; 1 0; 0 0; 2 0; 3

Luego U (Z2 Z3 ) = 1; 1

Por otro lado, las tablas correspondientes para Z2 y Z3

0 1 2
0 1
0 0 0 0
0 0 0
1 0 1 2
1 0 1
2 0 2 0

Luego, U (Z2 ) = 1 y U (Z3 ) = 1 :


As, resulta claro que U (Z2 ) U (Z3 ) = 1; 1 = U (Z2 Z3 )

7.2.3 Denicin. Sea A un anillo. Se llama divisor de cero a un elemento


x 2 A tal que xy = 0A para algn y 2 A :

Ejemplo.
Considere la multiplicacin en Z4

0 1 2 3
0 0 0 0 0
1 0 1 2 3
2 0 2 0 2
3 0 3 2 1

Podemos observar que


2 2=0
luego 2 es un divisor de cero en Z4 :

7.2.4 Denicin. Se llama dominio de integridad a un anillo unitario y con-


mutativo sin divisores de cero.

7.2.1 Teorema. El conjunto Z [i] ; bajo la adicin y multiplicacin habituales


de C forma un dominio de integridad.

102
Demostracin. La terna (Z [i] ; +; ) resulta efectivamente ser un anillo con
las operaciones heredadas del anillo C; adems se tiene que 1A = 1 + 0i 2 Z [i] y
verica que 8z 2 Z [i] ; 1A z = z: Finalmente es claro que Z [i] no tiene divisores
de cero, puesto que si hubiese tendriamos que z w = 0 con z y w 6= 0; luego
si z = a + bi y w = c + di con b y d no simultneamente nulos; su producto es
(ac db) + (cb + ad) i = 0 de aqu que (ac db) = (cb + ad) i; pero esto pasa
solo si consideramos la parte imaginaria igual a cero.

7.2.5 Denicin. Se dice que A es un dominio eucldeo (DE) si existe una


aplicacin
kk:A !N

tal que:

i. kxk = 0 si y slo si x = 0
ii. kxyk = kxk kyk
iii. Si x; y 2 A ; existe r 2 A; tal que yjx r y krk < kyk :

7.2.2 Teorema. Z [i] es un anillo eucldeo, con kzk = a2 + b2 :

Demostracin.

i. Es claro que si z = 0 + 0i; entonces kzk = 0:


Ahora, si kzk = 0 =) ka + bik = 0; por denicin de kk ; tenemos a2 + b2 = 0;
lo cual ocurre si a y b son cero simultneamente. i:e; z = 0 + 0i:
ii. Tomemos z = a + bi y w = c + di; luego zw = (ac db) + (cb + ad) i;
2 2
entonces kzwk = k(ac db) + (cb + ad) ik = (ac db) + (cb + ad) =
a2 + b2 c2 + d2 = kzk kwk :
iii. Para terminar el teorema procedemos como sigue

z a + bi (a + bi) (c di) ac + bd ad + bc
= = = 2 + 2 i
w c + di c2 + d2 c + d2 c + d2

Aplicando el algoritmo de la divisin resulta


1 2
ac + bd = q1 c2 + d2 + r1 ; jr1 j c + d2 ; q1 ; r 1 2 Z
2
1 2
ad + bc = q2 c2 + d2 + r2 ; jr2 j c + d2 ; q2 ; r 2 2 Z
2

103
Luego podemos escribir
z r1 + r2 i
= 2 + (q1 + q2 i) ;
w c + d2
de donde obtenemos
r1 + r2 i
r= y=x (q1 + q2 i) y 2 Z [i]
c2 + d2
Este elemeto r 2 Z [i] y verica que yjx r: Finalmente calculamos krk y
obtenemos
r2 + r22
krk = 1 2 kwk
(c2 + d2 )
como elegimos

1 2 jri j 1
jri j c + d2 ; entonces 2
2 (c + d2 ) 2

luego
1 1 1
krk + kwk = kwk < kwk :
4 4 2

7.2.6 Denicin. Sean x; y 2 Z [i] : Se dice que z 2 Z [i] es:


i. Un mximo comn divisor (mcd) de x; y si z divide tanto a x como a y, y es
mltiplo de cualquier otro divisor de ambos.
ii. Un mnimo comn mltiplo (mcm) de x; y si z es mltiplo de x y de y; y
divide a cualquier otro mltiplo de ambos.

7.2.7 Denicin. En el anillo eucldeo Z [i] un elemento que no sea una


unidad se dice que es un elemento primo de Z [i] siempre que = zw
donde z y w estn en Z [i] ; se tiene que uno de los dos z w es una
unidad en Z [i] : Un elemento primo es un elemento en Z [i] que no puede
ser factorizado en Z [i] en forma que no sea trivial.

7.2.1 Proposicin. Si es un elemento primo en el anillo eucldeo Z [i] y jzw


donde z; w 2 Z [i] ; entonces divide al menos a uno de los elementos z
w.

104
7.2.3 Teorema. Factorizacin nica. Sea Z [i] el anillo eucldeo y z 6=
0 un elemento de Z [i] que no es una unidad. Supongamos que z =
0 0 0 0 0
1 2 3 n = 1 2 3 m donde los i y los j son elementos pri-
mos de Z [i] : Entonces n = m y cada i ; 1 i n es un asociado de
algn 0j ; 1 i m y recprocamente, cada 0j es un asociado de algn
i:

0 0 0
Demostracin. Fijmonos en la relacin z = 1 2 3 m = 1 2 3 n
0 0 0 0
1j 1 2 3 n ; de donde 1 j 1 2 3 m : Por la proposicin 7.2.1 1 debe
dividir a algn 0j ; como 1 y 0j son ambos elementos primos de Z [i] y 1 j 0j ,
ambos elementos deben de ser asociados y 0j = u1 1 donde u1 es unidad de
0 0 0 0 0 0 0 0
Z [i] :Tenemos, pues, 1 2 3 n = 1 2 3 m = u1 1 2 i 1 2 m:
Repitiendo el razonomiento sobre esta relacin 2 ; y as sucesivamente, despus
de n pasos el primer miembro se hace 1; y el segundo un producto de un cierto
nmero de 0 (el exeso de m sobre n). Esto obligara a que n m ya que las
0
no son unidades. Anlogamente, m n; de modo que n = m: Y a lo largo
del proceso demostrativo hemos probado tambin que cada uno de los i tiene
algn 0j como asociado y recprocamente.

Al combinar la proposicin 7.2.1 y el teorema 7.2.3 tenemos que todo ele-


mento distinto de cero en el anillo eucldeo Z [i] puede ser escrito en forma nica
(salvo asociaciones) como un producto de elementos primos o es unidad en Z [i] :

7.2.8 Denicin. Un dominio de factorizacin nica (DF U ) es un dominio


de integridad en el que se cumplen
i. Todo elemento irreducible es primo
ii. Todo elemento que no sea unidad es producto de elementos irreducibles.

Ntese que como Z [i] es un anillo eucldeo y por teorema 7.2.3, Z [i] resulta
ser un dominio de factorizacin nica.

Las intenciones de esta discusin sobre la naturaleza de Z [i] ; p


es ilustrar las
deniciones, adems en adelante ser necesario trabajar sobre Z 3 pero no
discutiremos su naturaleza, pero
p usaremos algunas de sus caratersticas, as el
lector podr caracterizar a Z 3 siguiendo el ejemplo de Z [i] :

105
6.3 Algunas Ecuaciones Diofnticas

Ternas Pitagricas.

Diofanto trat en su Aritmtica el problema de encontrar ternas de nmeros


naturales no nulos x; y; z tales que x2 + y 2 = z 2 : Estas son llamadas pitagricas
porque segn el teorema de Pitgoras permiten construir tringulos rectngulos
con lados enteros. En este apartado vamos a encontrar todas las ternas que
satisfacen a la ecuacin x2 + y 2 = z 2 ; que corresponde al caso n = 2; en la
ecuacin xn + y n = z n :

Primeramente consideremos la terna (x; y; z) pitagrica, entonces tambin


lo ser (nx; ny; nz) para cualquier nmero n; y recprocamente, dada una terna
pitagrica (nx; ny; nz) ; podemos dividir sus componentes por su mcd para
obtener otra que cumpla adems (x; y; z) = 1: Una terna cuyo mcd es 1 se
llama primitiva. Si encontramos un mtodo para encontrar las ternas primiti-
vas, las restantes se obtienen multiplicndolas por nmero arbitrarios, luego el
problema estar resuelto.

Observemos que un divisor primo de dos de las componentes de una terna


pitagrica, divide a la tercera. Por ejemplo, pjz ^ pjy; entonces pjz 2 y 2 ; con
lo que pjx2 por lo tanto pjx: Esto signica que, en realidad, las componentes de
una terna pitagrica primitiva son primas entre s dos a dos, ntese que adems
es cierto que, en una terna pitagrica hay siempre dos componentes impares y
una par.

Ahora veamos que z ha de ser par. Consideremos lo contrario, esto es, x ,y


impares, x = 2m + 1; y = 2n + 1;
luego
x2 = 4m2 + 4m + 1; y 2 = 4n2 + 4n + 1
Al considerar clases mdulo 4 resulta que

z 2 = x2 + y 2 = 1 + 1 = 2

2 2 2
Pero ninguna clase mdulo 4 tiene a 2 por cuadrado: 0 = 0; 1 = 1; 2 = 0;
2 2
3 =1

Podemos suponer que x es par e y impar. Segn lo dicho z es tambin impar.


Consecuentemente z + y, z y son ambos pares. Pongamos x = 2u; z + y = 2v;
z y = 2w:
Ahora x2 = z 2 y 2 = (z + y) (z y) ; luego u2 = vw; v > 0; w > 0:

106
Por otra parte (v; w) = 1; ya que si un primo p divide a ambos, entonces
1 1 1
pj (v + w) = (z + y) + (z y) = 2z = z;
2 2 2
1 1
pj (v w) = (z + y) (z y) = y;
2 2
y como (y; z) = 1; esto es contradictorio.

Por la factorizaci nica en Z; es claro que si vw = u2 con (v; w) = 1; v > 0;


w > 0; entonces tanto v como w han de ser cuadrados. Pongamos v = p2 y
w = q 2 : Es claro que (p; q) = 1:

As tenemos que

z = v + w = p2 + q 2 ; y = v w = p2 + q 2 : En particular q < p

Como z e y son impares, p y q deben tener paridad opuesta. Sustituyendo


en las frmulas anteriores obtenemos
2
x2 = z 2 y 2 = p4 + 2p2 q 2 + q 4 p4 + 2p2 q 2 q 4 = (2pq) ;

luego x = 2pq: En consecuencia la terna original queda de la forma

(x; y; z) = 2pq; p2 q 2 ; p2 + q 2 ;

donde p; q son nmeros naturales primos entre s, q < p y de paridad opuesta.


Por lo tanto ya sabemos enumerarlas todas.

En la siguiente tabla mostraremos las ternas para los valores de p 7

p q x y z
2 1 4 3 5
3 2 12 5 13
4 1 8 15 17
4 3 24 7 25
5 2 20 21 29
5 4 40 9 41
6 1 12 35 37
6 5 60 11 61
7 2 28 45 53
7 4 56 33 65
7 6 84 13 85

107
7.3.1 Teorema. La ecuacin, x4 + y 4 = z 2 no tiene soluciones enteras posi-
tivas.

Demostracin. Si existen soluciones a la ecuacin x4 + y 4 = z 2 ; entonces


x ; y 4 ; z 2 es una terna pitagrica. Si dividimos x; y; z por su mcd obten-
4

emos coprimos que siguen cumpliendo la ecuacin, luego podemos suponer que
(x; y; z) = 1; y claramente esto implica que en realidad son coprimos dos a dos
y que la terna x2 ; y 2 ; z es primitiva.
Segn los resultados de la seccin anterior, x2 = 2pq; y 2 = p2 q 2 ; z = p2 +q 2
donde p y q son nmeros enteros coprimos, de distinta paridad y p > q > 0
(intercambiamos x con y si es necesario para que x2 sea el par)

Ahora, p2 = y 2 + q 2 ; luego (q; y; p) es otra terna pitagrica, lo que obliga a


que p sea impar, luego q ha de ser par, y as q = 2ab, y = a2 b2 ; p = a2 + b2 ;
para ciertos enteros a y b coprimos, de paridad opuesta, a > b > 0 (ntese que
se trata de una terna primitiva porque (p; q) = 1).
2
Por lo tanto x2 = 4ab a2 + b2 y en consecuencia ab a2 + b2 = x2 : Por
otra parte (a; b) = 1 implica fcilmente que ab; a2 + b2 = 1: Si el producto de
dos nmeros naturales coprimos es un cuadrado, entonces ambos son cuadrados,
pues cada uno de ellos debe tener cada factor primo con exponente par.

Concluimos que ab y a2 + b2 son cuadrados y, por el mismo argumento,


tambin lo son a y b: Digamos a = u2 ; b = v 2 ; a2 + b2 = w2 :
Entonces u4 + v 4 = a2 + b2 = w2 = p < p2 + q 2 = z < z 2 :

En resumen, si existe una terna de nmeros positivos (x; y; z) de manera que


x4 + y 4 = z 2 ; existe otra (u; v; w) que cumple lo mismo pero con w2 < z 2 : Si
existieran tales ternas debera haber una con z mnimo, lo cual es falso segn
lo visto, por lo que la ecuacin no tiene solucin.

En particular el teorema anterior implica que la ecuacin x4 + y 4 = z 4 no


tiene soluciones enteras positivas.

7.3.2 Teorema. La ecuacin, x4 + y 4 = z 4 no tiene soluciones enteras posi-


tivas.

Demostracin. Si fuese cierto x4 + y 4 = z 4 ; para ciertos enteros positivos


x; y; z; entonces, deniendo = z 2 ; tendramos que x4 + y 4 = 2 ; lo cual por el
teorema 7.3.1 sabemos que no puede ser, as la ecuacin x4 + y 4 = z 4 no tiene
solucin.

108
7.3.3 Teorema. No exiten enteros positivos x; y; z tales que

x3 + y 3 = z 3

Demostracin. Supongamos que existen nmeros (x; y; z) que cumplen


x3 + y 3 = z 3 : Dividindolos entre su mcd podemos suponer que son primos
entre s y, al cumplir la ecuacin deben ser primos dos a dos. Es claro que a lo
sumo uno de los tres puede ser par, pero si x e y son impares z es par, luego
exactamente uno de ellos es par.

Por simetra podemos suponer que x e y son impares. Entonces x + y, x y


son pares, digamos x + y = 2p; x y = 2q: As x = p + q, y = p q:
Consideremos la factorizacin

x3 + y 3 = (x + y) x2 xy + y 2

Sustituyendo obtenemos

2 2
x3 + y 3 = 2p (p + q) (p + q) (p q) + (p q) = 2p p2 + 3q 2

Podemos armar que p y q son coprimos (un factor comn lo sera tambin
de x; y) y tienen paridad opuesta (porque x = p + q es impar) : Cambiando el
signo de x; y; z si es necesario podemos suponer que x + y > 0, luego p > 0 e
intercambiando x con y si es necesario, tambin q > 0 (no puede ser que x = y,
pues q sera 0, y como (x; y) = 1 habra de ser x = y = 1, y entonce z 3 = 2; lo
cual es imposible).

Resumiendo, si existe una solucin (x; y; z) con x e y impares, entonces


existen nmeros naturales no nulos p y q de paridad opuesta, coprimos tales
que el nmero 2p p2 + 3q 2 es un cubo.
Debemos de justicar que los nmeros 2p y p2 + 3q 2 son coprimos, con lo
que cada uno ser un cubo. Notemos primero que, como p y q tienen paridad
opuesta, p2 + 3q 2 es impar, de donde se sigue claramente que 2p; p2 + 3q 2 =
p; p2 + 3q 2 = p; 3q 2 y como (p; q) = 1 el nico factor comn de p y 3q 2 es 3:
En otras palabras, si 3 - p; entonces 2p; p2 + 3q 2 = 1: Supongamos que es as.

Entonces, segn lo dicho, 2p y p2 + 3q 2 son cubos. Ahora usaremos un


resultado que posteriormente probaremos.

7.3.1 Proposicin. Si los enteros p; q; r cumplen p2 + 3q 2 = r3 ; (p; q) = 1


y r es impar, entonces existen enteros a y b tales que p = a3 9ab2 ;
q = 3a2 b 3b3 ; r = a2 + 3b2 :

109
La prueba de esta proposicin se dar ms adelante.

Admitiendo esto, p = a (a 3b) (a + 3b) ; q = 3b (a b) (a + b) :Claramente


a y b son coprimos y tienen paridad opuesta (de lo contrario p y q seran pares) :
Por otra parte 2p = 2a (a 3b) (a + 3b) es un cubo. Veamos de nuevo que
los factores 2a, a 3b y a + 3b son coprimos dos a dos, con lo que los tres sern
cubos.

Como a y b tiene paridad opuesta, a 3b y a + 3b son impares, luego un


factor comn de 2a y a 3b es un factor de a y a 3b; y por tanto un factor
comn de a y 3b: Igualmente un factor comn de a + 3b y a 3b lo es de a y 3b;
luego basta probar que (a; 3b) = 1: Puesto que (a; b) = 1; lo contrario obligara
a que 3ja; pero entonces 3jp y estamos suponiendo lo contrario.

As pues, 2a = u3 ; a 3b = v 3 ; a + 3b = w3 ; luego v 3 + w3 = 2a = u3 :
Nuestro objetivo es encontrar una solucin de la ecuacin de Fermat con z 3
par y menor que el valor del que hemos partido. As podremos concluir que no
pueden existir tales soluciones ya que no puede haber una mnima. Hemos de
reordenar la terna (u; v; w) para dejar en tercer lugar la componente par. Como
u3 v 3 w3 = 2a (a 3b) (a + 3b) = 2pjz 3 ; lo cierto es que la componente par, sea
cual sea, es menor en mdulo que z 3 :

Falta llegar a la misma conclusin si 3jp: Supongamos que p = 3s y que


3 - q: Entonces nuestro cubo es 2p p2 + 3q 2 = 32 2s 3s2 + q 2 y los nmeros
32 2s y 3s2 + q 2 son coprimos, pues (s; q) = 1 obliga a que los nicos divisores
comunes posibles sean 2 y 3; pero 3s2 + q 2 es impar (luego 2 no sirve) y 3-
q; (luego tampoco sirve) :

Consecuentemente 32 2s = u3 y 3s2 + q 2 = v 3 : Por la proposicin 7.3.1


llegamos a que q = a (a 3b) (a + 3b) ; s = 3b (a b) (a + b) : Por otro lado
32 2s = 33 2b (a b) (a + b) es un cubo, luego 2b (a b) (a + b) tambin lo es.
Luego hemos encontrado una terna menor que (x; y; z) que satisface la ecuacin
lo cual no es posible puesto que (x; y; z) es primitiva.

Para nalizar probaremos la proposicin 7.3.1


p
Demostracin. Consideraremos la factorizacin en Z 3 ; aqu los ele-
mentos de norma impar se descomponen de forma nica en producto de primos,
y esto es suciente
p para nuestros propsitos. En efecto, como (p; q) = 1 el
nmero p + q 3 no es divisible entre enteros no unitarios , es decir, no es
divisiblepentre primos que se conservan,
p y si un primo p = 1 2 se separa y
1 jp + q 3; entonces 2 - p + q 3: Por lo tanto la descomposicin en primos
es
p n1 nr
p+q 3= 1 r ;

110
donde N ( i ) = pi son primos distintos dos a dos. Tomando normas queda
que
r3 = p2n
1
1
p2n
r ;
r

p p
luego 3jni para todo i; lo que implica que p + q 3 es un cubo en Z 3 :
Por consiguiente
p p 3 p
p+q 3= a+b 3 = a3 9ab2 + 3a2 b 3b3 3;

y esto prueba la proposicin.

6.4 Ejercicios propuestos

1. Prubese el teorema 7.1.1


2. Prubese el teorema 7.1.2
p
3. Prubese que Z 2 es un anillo
p
4. Prubese que el anillo Z 2 es un dominio eucldeo.
5. Demustrese que las nicas soluciones enteras de la ecuacin y 2 + 3
p2 = x son
y = 5; x = 3: sugerencia. Considere la factorizacin en Z 2
p
6. Defnase y disctase todas las propiedades de Z 3
7. Demustrese que uno de los nmeros pitagricos correspondientes a estas
ternas debe ser mltiplo de 3.
8. Resulvase la ecuacin x2 + 2y 2 = z 2 en el anillo de los nmeros enteros.

111
Apendice

112
7 Notas de lgebra

"El lgebra es generosa, frecuentemente da ms de lo que le piden"


DAlembert.

7.1 Estructuras Algebraicas

En esta seccin deniremos varias estructuras algebraicas y algunos elementos.


Teniendo como nalidad presentar una breve semblanza de algunas estructuras
algebraicas y as poder situar al lector en una posicin ms comoda a la hora
de realizar la lectura del documento.

7.1.1 Denicin. Sea un conjunto no vaco. Llamaremos ley de composi-


cin interna (o simplemente ley de composicin) denida sobre a toda
aplicacin:

: !
Si x; y 2 escribimos

(x; y) ! (x; y)

Al elemento x y lo denominamos la composicin (por ) de x con y.


La denicin que acabamos de dar, es la formalidad para una operacin
binaria. Las operaciones de suma y producto entre nmeros constituyen los
ejemplos naturales de leyes de composicin.

7.1.2 Denicin. Se llama Monoide a toda pareja ( ; ) formada por un


conjunto 6= ; y una ley de composicin : Diremos tambin que
dene sobre una estructura de monoide.

Por ejemplo:
Si denota cualquiera de los conjuntos numricos ya conocidos

N; Z; Q; R; C
y denota la suma (+) o el producto ( ) habituales para estos nmeros, las
siguientes parejas forman monoides:

113
(N; +) (N; )
(Z; +) (Z; )
(Q; +) (Q; )
(R; +) (R; )
(C; +) (C; )

7.1.3 Denicin. Se denomina Semigrupo a todo monoide cuya ley de com-


posicin es asociativa.

En otras palabras,

x; y; z 2 =) x (y z) = (x y) z
Por ejemplo.
Todos los monoides numricos considerados en la denicin anterior:

(N; +) ; (N; ) ; (Z; +) ; (Z; ) ; (Q; +) ; (Q; ) ; (R; +) ; (R; ) ; (C; +) ; (C; )

forman semigrupos.

7.1.4 Denicin. Un Grupo es una pareja ( ; ) donde es un conjunto no


vaco y

: !
es una operacin binaria

(x; y) ! (x; y)

Tal que

i. x (y z) = (x y) z
ii. Existe un elemento e 2 ; llamado elemento de identidad, tal que: 8x 2
;x e = x = e x
iii. Para cada x 2 existe un elemento, llamado inverso, denotado con x;
tal que:(x) ( x) = e
iv. Diremos que el grupo es conmutativo o abeliano si satisface: x y = y
x; 8x; y 2

Las siguientes duplas forman grupos conmutativos:

(Z; +) ; (Q; +) ; (Q; ) ; (R; +) ; (R; ) ; (C; +) ; (C; )

Las duplas (N; +) ; (N; ) y (Z; ) no forman grupos, puesto que carecen de
elemento inverso para la operacin respectiva.

114
7.1.5 Denicin. Un Anillo es una terna ( ; ; ) donde es un conjunto no
vaco, y son operaciones binarias tales que
i. ( ; ) es un grupo conmutativo
ii. ( ; ) es un semigrupo

iii. x (y z) = (x y) (x z), 8x; y; z 2 distributividad de con respecto


a

Las siguientes ternas forman anillos: (Z; +; ) ; (Q; +; ) ; (R; +; )

Si un anillo ( ; ; ) satisface

iv. ( ; ) es un semigrupo conmutativo, entonces ( ; ; ) se llamar anillo


conmutativo.

Si ( ; ) es un semigrupo con identidad, diremos que ( ; ; ) se llamar


anillo con identidad.

7.1.6 Denicin. La terna ( ; ; ) es un cuerpo si y slo si es un anillo


conmutativo, con unidad, cuyos elementos no nulos admiten inverso mul-
tiplicativo. Esto es
i. ( ; ) es un grupo abeliano
ii. ( f0g ; ) es un grupo abeliano
iii. a (b c) = a b a c; 8a; b; c 2 :

Son ejemplos de cuerpos: (Q; +; ) ; (R; +; ) ; (C; +; ) ; (Zp ; +; )

Elementos distinguidos de un conjunto ordenado.

Sea un conjunto ordenado por una relacin de orden <

i. Primer elemento. El elemento a 2 se llama primer elemento si y slo si


precede a todos los dems.

a2 es primer elemento () x 2 =) a < x

ii. ltimo elemento. El elemento b 2 se llama ltimo elemento si y slo si


todo elemento de precede a b:

b2 es ltimo elemento () x 2 =) x < b

115
iii. Elemento mnimo. El objeto m de es elemento mnimo si y slo si no
existe un elemento distinto que lo preceda

m2 es elemento mnimo () 8x 2 : x < m =) m = x

iv. Elemento mximo. El objeto n de es elemento mximo si y slo si no


existe un elemento distinto que lo siga

n2 es elemento mximo () 8x 2 : n < x =) x = n

La funcin factorial y los nmeros combinatorios

7.1.7 Denicin. La funcin factorial es la aplicacin

f : N0 ! N

denida por
8
< f (0) = 1
f (1) = 1
:
f (h + 1) = (h + 1) f (h) si h > 1

La notacin ms usual para esta aplicacin es h!: De este modo lo anterior


se traduce en
8
< 0! = 1
1! = 1
:
(h + 1)! = (h + 1) h!
La expresin h! se lee "h factorial"

Propiedades del factorial

i. n! = 1 2 3 n
n 1 1
ii. (n+1)! = n! (n+1)!

7.1.8 Denicin. Sean los enteros no negativos n y k; tales que n k: Lla-


n
mamos nmero combinatorio "n sobre k" , al smbolo denido por
k

n n!
=
k k! (n k)!

116
Propiedades de los nmeros combinatorios.

i. Dos nmeros combinatorios de rdenes complementarios son iguales

n n! n! n
= = =
k k! (n k)! (n k)!k! n k

ii. La suma de dos nmeros combinatorios no es, en general, un nmero com-


binatorio; pero si tiene igual numerador y denominador consecutivos vale
la frmula
n 1 n 1 n
+ =
k 1 k k

117
Respuestas a ejercicios de tipo numrico

Captulo 2.

a. verdadero
b. falso
1. c. verdadero
d. falso
e. falso

10. mcd (576; 73) = 1; x = 2, y = 71


11. 600 y 120
12. 168, 18
13. 576; 162
16. a) 31; 44 b) 3; 2 c) 7; 8
17. para pq es 4; la de p2 q es 6; la de p2 q 2 es 9; y la de pn q m es (m + 1) (n + 1)

Captulo 3.

1. 1; 1; 2; 3; 5; 8; 13; 21; 34; 55; 89; 144; 233; 377; 610; 987; 1597; 2584; 4181; 6765; 10946:
3. u1 + u3 + u5 + + u2n 1 = u2n
4. mcd (13; 34) = 1; mcd (5; 610) = 5; mcd (3; 1) = 1; mcd (mcd (10946; 144) ; mcd (13; 34)) =
2
35 1 237 1 285 1
8. 157 =0+ 4+ 1 ; 101 =2+ 2+ 1 ; 126 =2+ 3+ 1
2+ 1 1+ 1
1 1+ 1
17 7+ 4+ 1
1+ 1 2
3

547 1 15 1
232 =2+ 2+ 1 ; 131 = 8+ 1
1+ 1 1+ 1
3+ 1 2+ 1
1+ 1 1+ 1
7+ 1 3
2

Captulo 4.

1. a) Todo x tal que x [x] < 21 ; b) Todo x; c) Todos los enteros, d) Todo
1
x tal que x [x] 2;e) Todo x tal que 1 x < 10
9

3. 143
4. ' (666) = 216; ' (153) = 96; ' (16384) = 8192

6. n impar; 35; 39; 45; 52; 56; 70; 72; 78; 84; 90

118
7. n par
X
8. ' (d) = 6 + 12 + 18 = 36
dj1729

Captulo 5.

1. Z7
+ 0 1 2 3 4 5 6 0 1 2 3 4 5 6
0 0 1 2 3 4 5 6 0 0 0 0 0 0 0 0
1 1 2 3 4 5 6 0 1 0 1 2 3 4 5 6
2 2 3 4 5 6 0 1 2 0 2 4 6 1 3 5
3 3 4 5 6 0 1 2 3 0 3 6 2 5 1 4
4 4 5 6 0 1 2 3 4 0 4 1 5 2 6 3
5 5 6 0 1 2 3 4 5 0 5 3 1 6 4 2
6 6 0 1 2 3 4 5 6 0 6 5 4 3 2 1

Z8
+ 0 1 2 3 4 5 6 7 0 1 2 3 4 5 6 7
0 0 1 2 3 4 5 6 7 0 0 0 0 0 0 0 0 0
1 1 2 3 4 5 6 7 0 1 0 1 2 3 4 5 6 7
2 2 3 4 5 6 7 0 1 2 0 2 4 6 0 2 4 6
3 3 4 5 6 7 0 1 2 3 0 3 6 1 4 7 2 5
4 4 5 6 7 0 1 2 3 4 0 4 0 4 0 4 0 4
5 5 6 7 0 1 2 3 4 5 0 5 2 7 4 1 6 3
6 6 7 0 1 2 3 4 5 6 0 6 4 2 0 6 4 2
7 7 0 1 2 3 4 5 6 7 0 7 6 5 4 3 2 1

3. x 5 mod 12
6. 43
8. 1; 9; 3; 81; 243; 27
11. a) 9, b) 12, c) 1, d) 13
12. a) 26, 61, 96. b) 624 c) 1829
14. 1; 13; 10; 7; 4
15. 317
16. 17x3 0 mod 17
19. 983

119
Conclusiones.

Este trabajo es un estudio y una recopilacin de las principales deni-


ciones, propiedades y teoremas que corresponden a un curso clsico de teora de
nmeros, desarrollados en nivel universitario de nuestro pas.

A lo largo de la construccin de este trabajo, abordamos los nmeros enteros


naturales y sus propiedades de divisibilidad que bsicamente es lo fundamental
en la teora de nmeros, sin embargo tambin tratamos la solucin de un tipo
particular de ecuacin, a saber las ecuaciones diofnticas y adems vimos la
divisibilidad desde la perspectiva de la congruencia; intentando mostrar todo
esto desde una ptica ms amigable en cuanto a contenido y tratamiento didc-
tico de la teora. Adems, hemos incluido algunas secciones atractivas donde se
muestran algunas aplicaciones para los resultados mostrados.

Todo esto lo hemos hecho con el objetivo de contribuir a la formacin de


estudiantes universitarios que tienen que enfrentar este tipo de cursos, puesto
que el n de este tratado es ser usado como un dispositivo didctico que facilite
la comprensin de la teora, como medio de consulta y ejercitacin y como
material de divulgacin en general.

Sin embargo las matemticas crecen a un nivel acelerado y la teora de


nmeros no es la excepcin, por tal razn la contribucin de este material es
muy modesta, as teniendo esto presente instamos a estudiantes a profundizar
en el estudio e investigacin de esta rama de las matemticas, y a instituciones
universitarias y tcnicas a modernizar sus programas de estudios para abordar
dichos aspectos.

120
Bibliografa

Apostol. Tom M.. Introduccin a la teora analtica de nmeros. Editorial


Revert, S.A, 1980. Espaa.
Boyer. Carl B. Historia de la matemticas. Tercera reimpresin, 1994.
Alianza Universidad Textos. Madrid
Burton W. Jones. Teora de los nmeros. Primera edicin en espaol, marzo
1969. Editorial Trillas, S.A. Mxico, D.F.
Gentile Enzo R. Aritmtica Elemental. Programa regional de desarrollo
cientco y tecnolgico. Washington, D.C. 1985
Herstein. I.N. lgebra Abstracta. Edicin Original 1986. Grupo Editorial
Iberoamricana, S.A. Estados Unidos.
Niven y Zuckerman. Introduccin a la teora de los nmeros. Editorial
Limusa,1976. Mxico.
Rbnikov K. Historia de las matemticas. Traduccin al espaol, editorial
Mir, 1987. Mosc
Richman Fred. Number Theory: An introduction to algebra. 1971 by
Wadsworth Publishing Company. Belmont, California.
Rojo Armando O. Algebra I. Sptima edicin. Editorial El ateneo, Ar-
gentina.
Sominski I.S. Lecciones populares de matemticas, Mtodo de Induccin
matemtica. Segunda edicin 1985. Editorial Mir, Mosc.
Suse Marx / Ehard Krinke/ Wolfram Turke. Teora elemental de los nmeros:
Ecuaciones y Combinatoria. Editorial pueblo y educacin. La Habana, 1974.
Vinogrdov I. Fundamentos de la teora de los nmeros. Segunda reimpre-
sin 1987. Editorial Mir, Mosc.
Vorobiov N.N. Lecciones populares de matemticas, Nmeros de Fibonacci.
Editorial Mir, Mosc, 1974.

121
Webgrafa

References
[1] http://www.sectormatematica.cl/libros.htm

[2] http://www.google.com/images?hl=es&q=bonacci&wrapid

122

You might also like